Этого треда уже нет.
Это копия, сохраненная 17 февраля 2018 года.

Скачать тред: только с превью, с превью и прикрепленными файлами.
Второй вариант может долго скачиваться. Файлы будут только в живых или недавно утонувших тредах. Подробнее

Если вам полезен архив М.Двача, пожертвуйте на оплату сервера.
 .jpg534 Кб, 1920x1080
Глупых вопросов тред 423747 В конец треда | Веб
Тут и только тут любой может задать глупый вопрос, но не глупый ответ.
2 423751
>>3747 (OP)
Снова Здравствуй, /sci/, в прошлом треде я реквестировал литературу с описанием классификации данных по видам/типам с точки зрения исследователей (не программистов с их целыми, логическими, и т.д.), мне посоветовали мануал к лабвью, несколько полезных для себя вещей я там нарыл, но это всё же не то.

>Данные %измерениенейм%, X-граммы (динамограммы, рентгенограммы, и т.д.), видео/фото/звукозаписи для различных целей (например, фото детали для её анализа на наличие микротрещин), в таком духе. Не требую прямо-таки полную и исчерпывающую классификацию, очевидно что всего этого овер9000, но хоть что-нибудь.



То есть, допустим, вот есть у нас медицина, так как мы лечим человека, мы проводим диагностику какого-нибудь органа, допустим сердца, сюда может относиться кардиограмма, кровяное давление, и так далее. С этой точкиз зрения классификация может выглядеть так:

Медицина->диагностика заболеваний сердца->кардиограмма
Медицина->диагностика заболеваний сердца->данные измерения кровяного давления
Медицина->диагностика заболеваний сердца->рентгенограмма грудной клетки
Медицина->диагностика заболеваний сердца->данные узи сердца

(порядок может быть иным, иерархия тут не очевидна)

Или, для качалок ШСНУ:

Нефтегазовое оборудование->штанговая скважинная насосная установка->мониторинг состояния->ваттметрограмма
Нефтегазовое оборудование->штанговая скважинная насосная установка->мониторинг состояния->динамограмма

Это очень приблизительно (рентгенограмма грудной клетки может использоваться для диагностики других заболеваний), но суть должна быть ясна, нужен своего рода перечень (возможно в виде иерархической модели).

Своего рода аналогом в другой области тут являются фонды физических эффектов, навроде таких:
http://www.gumer.info/bibliotek_Buks/Science/ukazat/
http://www.metodolog.ru/00751/00751.html
3 423770
Свет искривляет пространство?
изображение.png3 Кб, 624x61
4 423771
>>3770
Да
5 423772
А пространство толкает объекты или просто растягивает их?
6 423774
Почему, когда я смотрю на отражение монитора в лакированной поверхности стола, оно кажется таким приятным, красочным и вызывает умиление? Дело в детальности или в свете, не могу понять?
7 423776
>>3772

>А пространство толкает объекты или просто растягивает их?


Разницы никакой. Это одно и то же.
8 423786
какова температура внутри Эвереста на уровне моря?
9 423787
а, и ещё, какова примерно могла быть максимальная высота гор на Земле в ходе геологических процессов?
>>3786
10 423793
>>3747 (OP)
Наследуемость гомосексуализма ~20%
https://www.ncbi.nlm.nih.gov/pubmed/18536986

Наследуемость гомофобии ~35%
https://www.ncbi.nlm.nih.gov/pmc/articles/PMC2292426/

как такое возможно?
11 423794
>>3793
Что не так?
12 423796
>>3794
Геями же рождаются, а гомофобия это всё культурное влияние.
13 423800
Попытался смоделировать частичное лунное затмение, которое было 7 августа. В stellarium Луна просто стала немного менее яркой, а в space engine тень земли вообще почему-то оказалась на верхней части Луны. Это программы кривые или я делаю что-то не так?
14 423808
>>3796
Мальчики рождаются с письками, которые им хочется засовывать в дырки. Иногда им нравятся не совсем те дырки, которые следовало бы по логике, но суть остаётся той же. Другое дело, когда мальчикам нравится ощущение какашки, ходящей взад-вперёд в своей заднице, которая при этом эту задницу ещё и разрушает потихоньку.

Гомофобия - это не культурное влияние, это как если бы говноеды называли тех, кто не любит говно, копрофобами. Так, выдуманный термин.
15 423809
>>3800

>stellarium Луна просто стала немного менее яркой


Стеллариум немного грешит в правдоподобности.

>в space engine тень земли вообще почему-то оказалась на верхней части Луны


У меня все нормально, возможно у тебя старая версия без нормального расчета эфемерид. Сейчас последняя 0.9.8.0e.
scr00003.jpg257 Кб, 1920x1018
16 423817
>>3809

> Стеллариум немного грешит в правдоподобности


Странно конечно, на первый взгляд он показался более реалистичным

> У меня все нормально, возможно у тебя старая версия без нормального расчета эфемерид. Сейчас последняя 0.9.8.0e.


Действительно, была 0.9.8.0, пропатчил до 0.9.8.0e, стало нормально. Спасибо
17 423821
Нейросети сейчас превосходят человека в задачах, которые раньше считались трудными для компьютеров: игра в го/шахматы/покер и т.п.

А есть ли перспективы приспособить нейросети к поиску доказательства математических теорем вроде как давно научились формализовать математические доказательства, или это неподходящее средство?
18 423822
>>3821
Нейроши подходящие. Но мат теоремы СЛИИИШКОМ асбтрактные. Может самые простые можно.
19 423823
Какими будут люди на Марсе ? Будет ли слабая гравитация влиять на организм человека и приводить к его мутациям?
20 423829
>>3747 (OP)
как происходт отражение на атомном(молекулярном) уровне. и можно ли увидеть на атоме 1/n изображения. у меня существуют подозрения, что нет...
но как тогда создается цепь из отражающих частиц?
раздел с физикой не нашел.....
это очень тупой вопрос?
21 423831
>>3829
Я отправлю тебя к книге фенмнана КЭД странная история вещества. Там очень доступно и без матана это пояснено.
22 423832
>>3831

>Фейнмана


фикс
23 423836
Телекинез существует? Если да, то какой механизм работы у него? Если нет, то почему?
24 423837
>>3836

>Телекинез существует?


Нет.

>Если нет, то почему?


Что значит почему? Его не существует потому же, почему не существует магии, телепатии, бога и прочей подобной хуиты, их выдумали люди, чтобы было не так скучно. Отсутствие чего-то не требует объяснения причины своего отсутствия.
25 423838
>>3837

>Нет.


Сиранул с этого учёного мужа, делающего авторитетные заключения на основе пустоты.
26 423839
>>3808

>которые им хочется засовывать в дырки


И сразу фейл.
27 423840
>>3837

>пук


Не надо при всех своё невежество показывать. Если не можешь объяснить почему?, то молчи и не позорься.
28 423844
>>3787
я полагаю не более 10 км, тупо из-за экзогенных процессов. на марсе например есть вулкан 22 км, но там нет атмосферы, соответственно нечему его разрушать
29 423846
Где достать по одной двояковогнутой и двояковыпуклой линзе с фокусами 40 мм и -40 мм соответственно и диаметром 25 мм? Или вообще любые какие-нибудь на заказ.
Ih-k8umQ8D4.jpg34 Кб, 720x409
30 423847
Увидел сейчас шутеечку в вк, и задумался. Можно ли теоретически разогнать корабль до сс, изпользуя ВСЮ энергию гепотетически бесконечной вселенной?
Даже не могу толком вопрос поставить, смогу я разогнать свой карабль на бумаге до 100%сс?
Или как ответить на пикрил?
31 423848
>>3847
Объект, имеющий массу, нельзя разогнать ровно до скорости света. Поэтому строй корабль из фотонов
изображение.png58 Кб, 1689x358
32 423849
Объясните этот абзац из вики:

> Обход классических ограничений был найден в 2006 году А. Коротковым и Э. Джорданом[51] из Калифорнийского университета за счёт слабых квантовых измерений (англ. weak quantum measurement). Продолжая аналогию, оказалось, что можно не распахивать ящик, а лишь чуть-чуть приподнять его крышку и подсмотреть в щёлку. Если состояние кота неудовлетворительно, то крышку можно сразу захлопнуть и попробовать ещё раз. В 2008 году другая группа исследователей из Калифорнийского университета объявила об успешной экспериментальной проверке данной теории. «Реинкарнация» кота Шрёдингера стала возможной. Наблюдатель А теперь может приоткрывать и закрывать крышку ящика, пока не убедится, что у наблюдателя Б кот окажется в нужном состоянии


Получается, при помощи запутанных частиц и слабых измерений всё-таки можно передать классическую информацию быстрее скорости света?
33 423850
>>3848
Не, хочу именно с массой, хотя бы до 0.9999999...9999, хотя бы как в той шутке про Ахиллеса и черепаху.
изображение.png2 Кб, 191x65
34 423851
>>3850
До 0.99... можно. Нужную энергию сам можешь посчитать
35 423854
>>3831
я тупой даун из Б.
я не буду читать целую книгу ради тупого вопроса
36 423856
Когда на одном из полушарий зима, это влияет на количество облаков ? Если да, то как и на какой части планеты?
37 423860
>>3854
Там это в самом начале. Ну да ладно. Тогда забей.
38 423861
>>3849

>Получается, при помощи запутанных частиц и слабых измерений всё-таки можно передать классическую информацию быстрее скорости света?


Нет. Не путай теплое и мягкое.
39 423869
>>3861
но ведь если на одной стороне постоянно проверять состояние запутанной частицы, то можно мгновенно узнать когда изменится состояние второй.
40 423870
>>3869
для этого как я понимаю, второй должен тебе сообщать своё состояние постоянно. И без этой инфы ты не сможешь как нужно "подсмотреть". Короче ты не можешь никак нарушать принципиальные постулаты. Они другим занимались немного.
41 423876
>>3838

> куча экспериментов опровергающих телекинез


> ни одного корректно поставленного эксперимента, подтверждающего его


> на основе пустоты


>>3840
Ну поясни тогда мне, почему на орбите между Землёй и Марсом не вращается чайник?
42 423887
>>3876

> куча экспериментов опровергающих телекинез


Ты долбоёб, что ли, вась?
Перед учёным сидит кролик. Учёный осматривает кроля, сравнивает его с таблицей и ставит галочку: "кролики существуют, был проведён опыт".
Перед тобой не сидит кролик. Ты делаешь ба-да-дум-тссс и записываешь: "кроликов не существует, опыта провести не смог ввиду отсутствия объекта".

Это тебя в школе такой науке научили?
43 423888
>>3887

>Перед тобой не сидит кролик. Ты делаешь ба-да-дум-тссс и записываешь: "кроликов не существует, опыта провести не смог ввиду отсутствия объекта".


Не таких экспериментов, дебс, ясен пень, что проверяли тысячи людей, которые утверждали, что владеют телекинезом или демонстрировали телекинез в проводимых ранее некорректно поставленных опытах.
44 423892
>>3888
Проверяем тысячу девушек, которые утверждают, что умеют сосать хуй. Все сосут плохо - делаем вывод, что хуй сосать никто не умеет!

Но как же твоя le maman? А она не пришла, была занята.
45 423893
>>3892
Ну да, все настоящие телекинетики специально не приходят на тестирование, интересно почему?
46 423894
>>3893
Потому что дебилов нет, вась. В XII веке всех сожгли уже. Дарвинизм.
47 423895
>>3894
ХХI век на дворе, мань, сейчас за демонстрацию паранормальных способностей дают миллион долларов, а не сжигают на костре. Но почему-то никто не хочет подзаработать на изичах.
48 423897
>>3895

>сейчас за демонстрацию паранормальных способностей дают миллион долларов, а не сжигают на костре


Хоть кому-то дали? Не? И какой же вывод ты из этого сделал?

Есть всего два варианта.
49 423898
>>3897

>Есть всего два варианта.


Нет, вариант всего один, потому что всё было юридически заверено при множестве свидетелей.
50 423900
>>3898
А, верующий в юриспруденцию в XXI веке. Бывает. Маме передавай, чтоб держалась.
51 423901
>>3900
Ну да, видные учёные тратят кучу бабок и всех наёбывают, только чтобы скрыть ужасную правду о юзлесс телекинезе милиграмма материи.
52 423902
>>3901

>видные учёные тратят кучу бабок


Учёные это не та профессия, которая ассоциируется с кучей бабок. Возможно, тебе нужно пересмотреть свои исходные позиции и задуматься, а откуда же всё-таки деньги.
53 423903
>>3902

>Учёные это не та профессия, которая ассоциируется с кучей бабок.


Неверно, современное лабораторное оборудование стоит очень немалых денег, а уж если вспомнить БАК...
54 423904
>>3903
Ты думаешь, их на зарплату строили?
55 423907
>>3904
Ты так и не объяснил зачем это делать? Какой смысл это скрывать?
7361900.jpg47 Кб, 807x566
56 423908
>>3894

> телекинетики существуют, просто не демонстрируют свои способности людям

57 423909
>>3907
Я не думаю, что это целенаправленно скрывают. Бритва Хайнлайна же. Не надо искать заговоров там, где жулики, бандиты, шоумены, пафос, телевидение.
Просто фонд существует до тех пор, пока он не выплачен. Его существование выгодно. Его выплата нет. Всосал?
>>3908

>телекинетиков не существует, потому что я ни одного не видел

58 423911
>>3909

>Его существование выгодно.


Это с какого перепуга-то? Они тратят на постановку экспериментов кучу бабок, гораздо больше самой выигрышной выплаты, и ничего не получают взамен.
59 423912
>>3911

>Они тратят на постановку экспериментов кучу бабок


Ооо да, шарики для пинг-понга и крутильные весы это ж столько бабла, ещё и расходуются каждый раз.

>и ничего не получают взамен.


Ты дурачок, что ли, вася? Они получают шоу. Вроде "Поля Чудес" или "Битвы шаманов".
60 423915
>>3912

>Они получают шоу.


Они не с самого начала транслировали это на большую публику, долгое время фонд существовал на свои средства.
61 423916
>>3909

>телекинетиков не существует, потому что я ни одного не видел


Я такого не писал. Лучше сказать: телекинетиков скорее всего не существует, потому что нет надёжных доказательств.
И ещё, по-моему разумно придерживаться принципа "Экстраординарные утверждения требуют экстраординарных доказательств". Поэтому одиночных исследований для подтверждения телекинеза явно мало. Вот если бы существовало несколько исследовательских центров в разных странах, с сотнями сотрудников, тщательно задокументированными экспериментами, тогда можно было бы признать существование телекинеза
62 423917
>>3916

>Вот если бы существовало несколько исследовательских центров в разных странах, с сотнями сотрудников, тщательно задокументированными экспериментами, тогда можно было бы признать существование телекинеза



Если бы они существовали, они были бы засекречены. /дискач.
63 423918
>>3917
По такой логике вся наука была бы засекречена.
64 423919
>>3918
А она засекречена. Всё, что может быть использовано в качестве оружия и для шпионажа. Ты не знал?
65 423920
>>3919
Ну и как телекинез может быть использован в качестве оружия или для шпионажа?
66 423922
Напомните, какие проводились эксперименты по определению односторонней скорости света?
67 423926
>>3916
Тут даже не "скорее всего" а "скорее скорее всего". Вероятность миллиардная доля от миллардной процента. Ибо нарушения законов современной физики надо умудриться провернуть. Ах да. люди с синдромом заговора никогда даже не заходили ни в одну лабу видимо, идите галоперидол выпейте.
68 423928
>>3919

>Ты не знал?


Нет. Пожалуйста, приведи пример "засекреченной науки" (не конкретной технологии, а именно принципиального эффекта)?
69 423929
>>3928
Бюджет РФ.
70 423930
>>3928

>Пожалуйста, приведи пример "засекреченной науки" (не конкретной технологии, а именно принципиального эффекта)?


А схуяли ты решил, что паранормальный эффект это не конкретная технология, а именно что-то принципиальное, няша? Тебе с Нибиру сигнал поступил?
>>3926

>нарушения законов современной физики


О, ну с этим нахуй сразу. Самолёт тяжелее воздуха, но летает, нарушает законы физики. Машина не жрёт сено, но едет, нарушает законы животноводства.
Смех2.webm394 Кб, webm,
360x356, 0:03
71 423931
>>3930

>О, ну с этим нахуй сразу. Самолёт тяжелее воздуха, но летает, нарушает законы физики.

image.png117 Кб, 801x848
72 423939
>>3931
Твёрдая, окончательная позиция современной науки 100-летней давности. Они провели тысячи убедительных опытов, и невозможность постройки самолёта была убедительно доказана.

Вот поэтому - сразу нахуй с такими аргументами.
73 423946
>>3930

>А схуяли ты решил, что паранормальный эффект это не конкретная технология, а именно что-то принципиальное, няша? Тебе с Нибиру сигнал поступил?


А схуяли ты решил, что вообще существуют эти паранормальный эффекты, если это секретная технология, няша? Тебе с Нибиру сигнал поступил?

Какие все таки магуны - дауны, неспособные к элементарной логике.
74 423947
>>3939
Надеюсь ты понимаешь чем наука 100 лет назад отличалась от современной?
75 423951
>>3946
Тебе прямой вопрос задан, а ты пытаешься сбросить беседу на начало. В который уже раз? В третий? Сколько раз тебе в рыло нассать за это?

Никто ИТТ не утверждал наличие чайников Рассела. Тебя ебут в анал за категорическое утверждение их отсутствия.
>>3947
Надеюсь, ты понимаешь, что ещё через 100 лет современная наука будет точно таким же источником лулзов.
76 423953
>>3947
Да, у науки был (и возможно есть) - консенсус по мнениям, которые оказалось ошибочными, но к счастью, наука - не так работает - научную истину не голосованием утверждают, представь. (Хотя консенсус экспертного сообщества стоит принимать во внимание, например, банкиру который хочет провести не рискованную инвестицию.)

>Твёрдая, окончательная позиция современной науки 100-летней давности.


Ты неумело врешь, дурашка.
Во первых, по этому поводу не было консенсуса (хотя, как я сказал - это так важно).
Во вторых, они говорили о нерешаемых (по их мнению) технических трудностях, для практически полезной реализации аппарата тяжелее воздуха (полет птиц то, они видели и никакой теории о особой био-левитации не придерживались).
77 423954
>>3953
Что сказать-то пытался?
78 423955
>>3951

>Сколько раз тебе в рыло нассать за это?


Астральной урины то хватит?
79 423956
>>3953
Фикс:

>это так важно


это не так уж важно
80 423960
>>3951

>Надеюсь, ты понимаешь, что ещё через 100 лет современная наука будет точно таким же источником лулзов.


Конечно, но принцип соотвествия не наебешь.
15096345510720[1].webm6,1 Мб, webm,
640x360, 1:56
81 423961
Матанобляди, поясните.

А в чем собственно отличие хуеты что он порет от перфоманса какой-нибудь обезьяны вроде Йоко Оно? Никто проверить правильность его слов не может, студенты организованы таким образом что они смотрят в рот и записывают чтобы потом воспроизвести, воспринимают все на веру. Потом в жизни они никак проверить хуйня это или нет все равно не смогут, для манагера по продажам формулы не требуются. Может математики просто придумывают это все, ну как гопобыдло прокачивает бредогенератор чтобы "грузить", вот и они так грузят чтобы была видимость сложности и непостижимости и деньги за это берут. Того кто научился так же складно пиздеть и прохавал фишку берут к себе шнырем аспирантом.
82 423963
>>3961
Это всё проверяется и на строгих правилах, так что не то же что и гопобыдло.
83 423965
>>3895

>ХХI век на дворе, мань, сейчас за демонстрацию паранормальных способностей дают миллион долларов, а не сжигают на костре.



>всё было юридически заверено при множестве свидетелей



Скотомразь в вашем лице особо не изменилась.
Со времён одного Пилата... Умывшего руки. Там вон тоже толпа народа была.
Всё та же лживая скотомразь. Может быть и хуже...

>Ты так и не объяснил зачем это делать? Какой смысл это скрывать?



Так вы тоже не объяснили зачем вам скрывать. Носитесь тут, гавкаете, что нет телекинеза... Это показывает что смысл должен быть >:-)
84 423966
>>3965
Ой я тебя таки умоляю, Иисус сам выбрал свою судьбу. Евангелие Иуды почитай.
85 423967
>>3965

>Скотомразь...


>... лживая скотомразь. Может быть и хуже...


>Носитесь тут, гавкаете, что нет телекинеза...


Как же мне доставляет боль и истерика маго-верунов.
Может мне стоит задуматься над своей реакцией - это же, наверное, плохо - так радоваться боли другого человека?
image.png41 Кб, 184x147
86 423968
>>3967
Это нормально, потому что наука это технически чёрная магия. Ты просто следуешь своим догмам. Хороший бобик.
87 423972
>>3965

>Со времён одного Пилата... Умывшего руки. Там вон тоже толпа народа была


а курочка Ряба несла золотые яйца
88 423974
>>3967

Угу. Мы не против доставить вам иногда. Удовольствие ответом. Так что можете сегодня порадоваться. Приходу. Ответа. :-)
89 423981
Я просто просил пояснить за телекинез. Если кто-то в штаны с этого насрал, то не надо размазывать его говно по всему треду. Достаточно адекватно ответить, а не как этот не видел знач нету)))>>3837
EffetTunnel.gif322 Кб, 200x200
90 423982
Может ли крупный объект, например человек, внезапно переместиться на какое-то значительное расстояние(пусть 1 метр) из-за туннельного эффекта? Понятно, что если это возможно, то вероятность где-то около нуля. Интересует принципиальная возможность такого случая
91 423983
На вики сказано, что для варп-драйва нужно 700кг антиматерии. Как они её будут использовать для исправления пространства?
92 423984
>>3983

>исправления


Искривления*
93 423985
>>3982
там такая вероятность, что она теряет всякий физический смысл. ты сформулировал саму задачу косо, дело в том что туннельный эффект, это не про моментальное перемещение,
это про прохождение над барьером
00676b5530d9[1].jpg18 Кб, 334x479
94 423986
есть такая теория что мир фрактален, и что микромир с атомами и электронами аналогичен(подобен) макромиру с планетами и звездами.
Так же известно что каждый атом что-то отражает/переизлучает(как минимум тепловое излучение). Возможно ли некой сущности, в глазом(матрицей датчиков) размером меньше электрона, "увидеть" эти излучения и получить "карту"(трехмерную модель) материи в довольно таки большом радиусе(метры, а то и километры) от себя так же как люди смотря в телескоп изучают Вселенную?
С одной стороны размер теплофотона довольной большой, с другой - атомы настолько далеко друг от друга что сквозь пространство между ними можно далеко "смотреть", так же как и в пространство между звёздами.
95 423987
>>3939

> Твёрдая, окончательная позиция современной науки 100-летней давности.


Только среди дебилоидов, даже тогда было очевидно, что птицы тяжелее воздуха, но летают.
96 423989
>>3981

> куча экспериментов опровергающих телекинез


> ни одного корректно поставленного эксперимента, подтверждающего его


> принципиальная физическая невозможность существования таких эффектов


> также эволюционная нестыковка (если бы такая способность существовала, то она закрепилась бы отбором)


> не видел знач нету)))

97 423990
>>3989

> также эволюционная нестыковка (если бы такая способность существовала, то она закрепилась бы отбором)


Ты хоть понимаешь что говоришь? Или просто цитируешь кого-то?
98 423991
Кстати, я так и не понял. А что вы вообще имеете ввиду под "телекинезом"?
99 423992
>>3990
Естественно понимаю, я ж не ты. Я закончил биологический факультет и сейчас работаю в научной сфере. Я отлично знаю как работает эволюция. Если тебе есть, что возразить по существу, то выкладывай. Почему этот признак не должен был закрепиться отбором?
100 423993
>>3991
Ну тип поднимать/двигать предметы силой мысли.
101 423994
>>3992

>Почему этот признак не должен был закрепиться отбором


Если я тебе про колесо расскажу, то ты так же меня спросишь почему эволюцией не закреплено!?
102 423995
>>3993
А что такое сила мысли?
103 423996
>>3993
Хорошо, на каком фундаментальном взаимодействии из четырех это должно работать? Эм? Гравитация?
104 423997
>>3996
Электромагнитное. Разве не очевидно?
105 423998
>>3997
Ну... я бы сказал, мол "очевидно что не на каком". Но допустим электромагнитное. Хорошо, откуда такое поле взять сильное? Какой орган будет создавать? Откуда энергию для поля брать?
106 423999
>>3994
То есть ты считаешь, что телекинез - это продукт цивилизации? Но мозговых имплантов, генерирующих гравитационные поля ещё не придумали, каким образом эта способность была сгенерирована цивилизацией?
>>3995
Ну тип сильно захотел, чтобы предмет подвигался, приложил волевое усилие - и он подвигался.
>>3996
По идее да, но хз как организм может её генерировать, в нём нет таких механизмов.
>>3997
Вряд ли, телекинитеки утверждают, что могут и диамагентики левитировать.
107 424000
>>3998

>Хорошо, откуда такое поле взять сильное? Какой орган будет создавать? Откуда энергию для поля брать?


Какой-то новый вид троллинга. Мне тебе теоретические расчёты и анатомическое строение рассказать?
108 424001
>>3999

>То есть ты считаешь, что телекинез


Я тебе говорю, что если ты в природе чего то не наблюдаешь здесь и сейчас это не значит, что чего-то нет. Возможно телекинез оказался энергетически не выгодным и обезьяна срывавщая банан рукой дольше оставалась сытой, чем обезьяна срывавшая тот же банан силой мысли.
109 424002
>>4000
Да почему, просто надо же объяснить как телекинез работает.А то я сколько видел, всё не задумывался, как они это делают.
110 424003
>>4001
Что мешает обезьяне-телекинетику срывать бананы руками, а телекинезом например удивлять тянок, тем самым получая преимущество в половом отборе? Телекинез ведь не имеет никаких побочных эффектов, чтобы отсеиваться отбором, одни плюсы.
111 424004
>>4003

>обезьяне-телекинетику срывать бананы руками, а телекинезом например удивлять тянок


Телекинез не обязательно должен быть как у Кэри или Юрико Омега. Возможно сорвать банан и есть предел сил. Но окупится ли израсходованная энергия съеданием того банана? Если нет, то телекинез не выгоден с точки зрения природы.
112 424005
>>4004
Применение телекинеза невыгодно, а не наличие. Кроме того есть куча способов более выгодно использовать телекинез, чем для срывания бананов, например, чистить плоды с колючей кожурой или вытаскивать всяких сусликов из нор.
113 424007
>>3987

> Кельвин


> Эдисон


> дебилоиды


А ты тогда кто? Хлебушек?
114 424009
>>4007
Мозг фрагментирован, гений в одном запросто может быть дебилоидом в другом.
warpdrive.jpg670 Кб, 2000x1110
115 424010
>>3983
Не антиматерия, а экзотическая материя, которая в отличии от первой только в теории существует
А насчёт вопроса - самому интересно, у меня только такая картинка есть
116 424011
>>4010
Нет, там речь именно об антиматерии, просто как удобной мере энергии.
>>3983
Они пока не знают, как искривлять пространство, просто посчитали теоретические затраты, при 100% КПД. Очевидно, что для искривления пространства нужна мощная гравитация, а вот как её создать, пока неизвестно. Хотя есть один эксперимент по созданию искусственной гравитации с помощью гравитомагнитного момента Лондона, там короче суть в том, что ускоренно вращающийся сверхпроводник генерирует гравитационное поле. Проблема в том, что спустя 11 лет этот эксперимент так до сих никто не пытался воспроизвести.
58a4das-960.jpg129 Кб, 960x960
Как измерить емкость аккумулятора? 117 424012
Все знают, что такое емкость? Ну например, показатель 60А/ч означает, что под нагрузкой 6А аккум разрядится за 10 часов.

Сейчас видел, как нагрузочной вилкой замеряют и объем, причем происходит это за 5 секунд, примерно.

Вот сбсн возник вопрос, как считает объем эта вилка? Можно ли узнать объем имея только мультиметр?
118 424013
>>4011
>>4010
А если такой корабль будет пролетать через какой-нибудь физический объект, то что произойдет? Будет ли столкновение или корабль пройдет сквозь него? Или от объекта оторвется кусок равный пространству внутри пузыря (там где корабль находится)?
119 424014
А пластины при эффекте Казимира отталкиваются или притягиваются?
120 424016
>>4013
Корабль не находится где-то в гиперпространстве, это не замкнутый пузырь, просто спереди пространство сжимается, сзади растягивается, если в эту переднюю сжимающуюся область попадёт какой-то объект, то его тоже попробует сжать, если он достаточно прочный, то не пострадает, так или иначе произойдёт обычное столкновение.
>>4014
Притягиваются.
121 424017
>>4016

>если он достаточно прочный, то не пострадает


А насколько нужно быть прочным, что бы не пострадать? Человек в том корабле пострадает? От сжатия.
122 424018
>>4017
Зависит от степени сжатия.
konk5.jpg152 Кб, 678x485
123 424019
>>3994

>почему эволюцией не закреплено!?


В некотором смысле - закреплено, только не в эволюции "генов", а в эволюции "мемов". Причем даже конкуренция была - те кто не мог или плохо мог делать "колеса" (в широком смысле) - были отпизжены и, или таки учились делать колеса или исчезли, в историческом смысле.

Вот и вопрос - ели "телекинез" возможен - где культуры/страны его использующие?
124 424020
>>4009

>Мозг фрагментирован, гений в одном запросто может быть дебилоидом в другом.


Кстати да, тот же лорд Кельвин - массу закидонов имел и довольно упрямый был (прям как некоторые скептики на Дваче, я в т.ч.).
125 424021
>>4012

>60А/ч


60 А·ч
126 424022
>>4019

>ели


если
бе.jpg50 Кб, 935x514
127 424023
>>4016
Что произойдет с сумкой на выходе, если она попадет в пузырь, но не столкнется с кораблем? Если она разрушится, то как именно? Просто из стороны в сторону покидает или и вовсе расщепит?
Что будет, если так же мимо планеты пролететь (т.е. без столкновения корабля непосредственно с планетой)?
>>4017

>Человек в том корабле пострадает


Корабль, как и его пилот, будут в ровненьком пространстве сидеть. Им ничего не угрожает.
128 424024
>>4018

>Зависит от степени сжатия.


Не троль меня!
А что вообще можно почитать доступного, о том, что происходит с материей при искривлении пространства? Или можешь в двух словах объяснить, не рвется же как лист бумаги?
129 424025
>>4023

>Корабль, как и его пилот, будут в ровненьком пространстве сидеть. Им ничего не угрожает.


Нет, тот корабль, мимо/сквозь который пролетает наш, в пузыре.
130 424026
>>4025
Значит я тот же вопрос задал, только у меня сумка, а у тебя корабль.
131 424027
>>4024

>Или можешь в двух словах объяснить, не рвется же как лист бумаги?


Если на примере растягивания (то есть позади корабля), то растягивает не за края, как рвут лист бумаги, а сразу во всех местах одновременно, если для разрыва силы не хватает, то он просто чуть-чуть удлинняется, а если хватает, то разрывается сразу на мономолекулярные полоски (при условии, что прочность одинаковая во всех местах).
132 424028
>>4027
Нихуя не понятно. Почему что-то должно произойти с материей, ведь расстояние между молекулами не меняется? Какое имеет значение прочность тела, ведь растягивается все одинаково?
133 424029
>>4028

>ведь расстояние между молекулами не меняется?


Как раз таки меняется, в этом и суть растягивания пространства.

>Какое имеет значение прочность тела, ведь растягивается все одинаково?


Не одинаково, всё растягивается в продольном направлении, а в поперечном никакого растягивания не происходит.
134 424030
>>4029

>всё растягивается в продольном направлении, а в поперечном никакого растягивания не происходит


То есть вперед тянется, а в высоту и ширину нет?
135 424031
>>4030
Именно!
136 424032
>>4031
То есть просто создав достаточной сжатости пузырь, то можно и черные дыры ломать?
137 424033
>>4032
Если скомпенсировать гравитацию чёрной дыры, то, по идее, она исчезнет, только хз, что будет с сингулярностью, тут современная физика пасует.
138 424034
>>4029

Но ведь, если, как ты говоришь, при искривлении пространства меняется расстояние межмолекулярное, то при малейшем существенном (от 10%) искривлении одинаково порвется и бумажный лист, и ваш титаниумовый коррабль.
>>4029

>Не одинаково, всё растягивается в продольном направлении, а в поперечном никакого растягивания не происходит.



...
139 424035
Блядь, хотел на б зайти, и промазал. Хуле тут ботаны одни сидят?
140 424036
>>4035
Да тут уже почти никто и не сидит, вот раньше были времена, нон-стоп срачи за вечные двигатели, свободную энергию, теорию всемирного кефира, нейросети. Переводчик был, эх...
141 424037
>>4035
Слыш да мы тут корабль построили, который может полететь в прошлое и дефлорировать твою мамку, до того, как это сделал твой дед. Так шо осодись ка ну.
142 424038
>>4034

>то при малейшем существенном (от 10%) искривлении одинаково порвется и бумажный лист, и ваш титаниумовый коррабль.


Не совсем, смотри, увеличивается расстояние между молекулами, действующие между ними связи создают обратно-направленную силу и молекулы стягиваются обратно, чем сильнее связь, тем быстрее молекулы будут стягиваться обратно и тем большая степень растяжения понадобится, чтобы окончательно их разорвать.
Screenshot2017-11-03-13-19-39.png61 Кб, 720x1280
143 424039
А почему оно вместе?
изображение.png122x36
144 424040
>>4039
В алфавитном порядке
145 424041
>>4037
БоТаН порвался, ггггггг)))))))))))))))))
146 424042
>>4041
Ладно, ботаны, пойду аниме смотреть, развиваться так сказать, всем пока!
147 424043
>>4038
А по моему вы хуйню несете, уважаемый. При растягивании/сжатии пространства расстояния не меняются, но это я так с дивана,я вообще про аккумулятор зашел спросить.
К сожалению не смог почему-то нагуглить инфы на эту тему, а не зная и спорить бесполезно.
148 424046
Аноны, какой учебник посоветуете по линейной и общей алгебре? Желательно с азов, чтоб освежить знания.
149 424047
>>4043

>При растягивании/сжатии пространства расстояния не меняются


Ты сейчас сказал, что 2=1, что при растягивании пространства оно не растягивается. Расстояния между телами - это есть главный определитель пространства, его суть.

>К сожалению не смог почему-то нагуглить инфы на эту тему


Вот инфа, например:
https://ru.wikipedia.org/wiki/Детектор_гравитационных_волн
Там в первом же абзаце написано про искажения пространства и изменение расстояний между частицами вследствие этого.
150 424050
>>4047
В таком случае ваш корабль хуйня и невозможен.
151 424052
>>4050

>В таком случае ваш корабль хуйня и невозможен


Почему?
1.png11 Кб, 591x495
152 424053
>>4047
Теперь непонятна другая хуйня, если мы растягивает область пространства, то за краями области пространство должно сжаться?

>>4052
Потому что в вашей теории фантазии нечему его двигать, кроме:

> Пузырь движется по неровностям пространства, как серфер на волне.


Откуда сверхсвет тут? А может оно будет двигаться 1 км в час, или не будет двигаться вообще.
153 424054
>>4053
А Ванину можно потеребить?
154 424056
>>4047

>Там в первом же абзаце написано про искажения пространства и изменение расстояний между частицами вследствие этого.



Конечно, но это растяжение относительно (более плоского пространства). И такое изменение расстояния "изнутри" заметить невозможно - нужно что бы база детектора выходила за размеры растянутой области (если растяжение равномерно). Никакие молекулы понятно, такого растяжения не заметят, если только частота такой гравитационной волны не будет такой высокой, что длинна волны станет сравнима с межмолекулярными расстояниями.
155 424057
>>4053

>Потому что в вашей фантазии нечему его двигать, кроме


Двигать будут генератор того самого искривления. С ним же и будет двигаться сама область искривления, что из-за своих свойств сократит расстояние.
Но чем сильнее искажение, тем меньше придется двигаться. Таким образом, при достаточном искажение, для путешествия в другую галактику нужно будет пролететь всего лишь метр.
156 424058
>>4056

>частота такой гравитационной волны не будет такой высокой, что длинна волны станет сравнима с межмолекулярными расстояниями


Но мы не о гравитационных волнах говорим.
157 424060
>>4057

> область искривления, что из-за своих свойств сократит расстояние


> написано про искажения пространства и изменение расстояний между частицами вследствие этого



Подожди, это ты мне пишешь, или разные люди?

> Двигать будут генератор того самого искривления. С ним же и будет двигаться сама область искривления



Откуда инфа эта? Почему ты решил, что оно, мало того, что будет двигаться, так еще и быстрее сс? Может это просто очевидно для тебя и пояснений не требует?

>>4054
Получается, что можно, лол.
1.png23 Кб, 946x514
158 424061
>>4053

>Теперь непонятна другая хуйня, если мы растягивает область пространства, то за краями области пространство должно сжаться?


С одной стороны мы сжимаем, с другой растягиваем, выглядит это вот так.

Двигать корабль будет обычный ракетный двигатель, вместе с ним будет двигаться и пузырь. Пузырь сократит расстояние до цели в миллионы раз, поэтому даже с обычным ракетным двигателем понадобится немного времени.

>А может оно будет двигаться 1 км в час


А может у бабушки вырастет хуй и она станет дедушкой? Все расчёты по охуевшим многоэтажным формулам проведены прошаренными физиками, которые писали статьи про пузырь, и по ним всё сходится.
>>4056

>И такое изменение расстояния "изнутри" заметить невозможно - нужно что бы база детектора выходила за размеры растянутой области


Так и есть, для этого длина плеча детектора сделана 4 км.
159 424062
+
И ты хочешь сказать, что находясь в определенной области пространства можно определить, искривлено оно иил в нормальном состоянии? Типа развел яруки в разные стороны, вроде норм, поворачивась на 90г, хуяк ладони скрылись в тумане космоса (свет от ладони я увижу с задержкой, относительно положения поперек искривления)?
160 424063
>>4060
Ну у корабля сзади будет пространство растягиваться, а спереди сжиматься. Двигаться будет генератор искажения - материальный объект. Просто именно за счет того, что он искривляет пространство, расстояние будет сокращаться.
Так что если ты достаточно исказишь пространство, то тебе нужно будет сдвинуться чуть вперед (например пернуть) и выключить генератор, что бы преодолевать сколь угодно большие расстояния.
161 424064
>>4062
Ага, если пространство растянуто не равномерно во все стороны, то можно определить по анизотропности.
163 424067
>>4058

>Но мы не о гравитационных волнах говорим.


А какая разница?
И вообще, там что, предполагается, такое сильное искривление, что на межмолекулярных расстояниях заметно?

>>4023
А разрушение тел в гравитационных полях, довольно хорошо изучено (в основном, расчетными методами, конечно) - "разрушение приливными силами" называется.
https://ru.wikipedia.org/wiki/Приливные_силы
(в астрономических делах еще и вращение включается)

Если градиент поля достаточно высок, что бы на разные концы "сумки" действовали достаточно разные силы - "сумку" разорвет конечно (если это дельта - будет больше прочности).

Помню, в какой-то детской научно-популярной книжке, была "миленькая" картинка с человечком разрываемым приливными силами при приближении к черной дыре.
164 424068
>>4066
Чё за ребусы начались? Объясняйся словами.
бе.jpg22 Кб, 838x506
165 424069
>>4066
Какой из серый отрезков короче: а или б ?
166 424070
>>4061

> пикрил


Лол, и у тебя пузырь будет проваливаться в более плотную область пространства? Ты ебанутый?
167 424071
>>4070
Ты хуйню какую-то написал. Как пузырь может проваливаться в более плотную область, если эта область - часть пузыря?
168 424072
>>4067

>Если градиент поля


Если градиент поля (что эквивалентно соответствующему искривлению)

>если это дельта


если эта дельта сил

Еще раз подчеркиваю - опасен только градиент, если поле полого, то человечек ничего не заметит (кроме синего смещения частоты внешнего сигнала).
169 424073
>>4069
Одинаковы, так как заключены в одинаковые области пространства, и имеют одинаковое расположение в оном.

>>4068
Луч проходит расстояние а - б за х. Как повлияет на результат х, размещение вблизи прохождения мульярда чугунных болванок ГОСТ 4832-95, с различных точек зрения?
изображение.png7 Кб, 500x200
170 424074
Вот короче более качественный рисунок пузыря, чем мои каракули, тут правда он не совсем как пузырь, но суть та же.
propulsionwarpdrive.gif1,6 Мб, 500x200
171 424075
>>4074
Блять, гифка сломалась.
172 424077
>>4075
>>4074
Так у тебя же просто нижний корабль быстрее двигается, вот и все. Зачем тогда шутить с пространством, если все равно решает скорость?
изображение.png5 Кб, 465x450
173 424078
>>4073

>Луч


Если речь идёт о световом луче, то он не пойдёт по траектории а1-б1, он повернёт из-за гравитационного линзирования и уже не вернётся на изначальный путь.
174 424079
>>4071
Ну возьми область (пузырь) в любой среде, создай уплотнение с одной стороны области, и разряженность с другой, куда сдвинется область?
175 424080
>>4073

>Одинаковы


Линейку приложи, дурень.
176 424082
>>4077
Нет, в своих субъективных пространствах они движутся с одинаковой скоростью. Верхнего корабля вообще могло не быть, он тут лишний, картинка лишь чтобы показать форму искривления пространства.
177 424083
>>4079
Область никуда не сдвинется сама по себе. Область создаётся генератором, и вот если сдвинуть генератор, то следом за ним сдвинется и область.
1t.jpg38 Кб, 500x297
178 424084
>>4080
Иди нахуй.
изображение.jpg16 Кб, 898x424
179 424085
Для стороннего наблюдателя расстояние не меняется. А для самого путешественника пункт А становится ближе к пункту Б за счет искривления самой дороги.
180 424086
>>4078
Да, действительно, тупанул.

>>4083

> и вот если сдвинуть генератор, то следом за ним сдвинется и область


И полетит с обычной скоростью, ибо на обратное ничего не указывает.
181 424087
>>4086

>И полетит с обычной скоростью


И? А кто-то говорит, что он будет быстрее лететь?
182 424089
>>4086

>И полетит с обычной скоростью


Не совсем, ведь перед ним уменьшенное количество пространства то есть меньшее расстояние, поэтому он преодолеет его быстрее. Конечно формально скорость та же, в этом и фишка, чтобы не превышать скорость света, но со стороны он будет выглядеть как очень быстро движущийся.
183 424131
>>4087
>>4089
Остается догадаться. почему вы решили, что "сжатое" пространство корабль пройдет быстрее обычного. А так получается, скорость в пространстве не меняется, "количество" пространства до цели не меняется.
Возьму для примера лес: ты стоишь в середине, и внезапно искривляешь лес таким образом, что оказываешься у самой его опушки, но перед тобой все те же деревья половины леса, только с плотностью бетона. Совершенно непонятно, как это поможет выйти из леса.
184 424132
>>4131

>"количество" пространства до цели не меняется.


В том то и дело, что меняется, сжать пространство=уменьшить его количество=уменьшить расстояние. Сжимание пространства даёт результат в виде уменьшения расстояния между точками, если бы этого не происходило, то в чём тогда выражается сжатие пространства, что меняется при сжимании?

>но перед тобой все те же деревья половины леса, только с плотностью бетона


Верно, но поскольку между звёздами вакуум, то его сжатие не мешает движению.
185 424133
>>4131
Какой лес? Какой бетон? Возьми две 2 печенья и 1 конфету.
Положи печенье друг на против друга, а конфету между ними. А теперь одно печенье отдали от конфеты, а второе на столько же приблизь.
В итоге, печенье, которое удалялилось от конфеты - пункт отправления (А), а печенье, что приблизилось, пункт назначения (Б). Получается, что ты расталкиваешь пространство между А и конфетой, и сжимаешь его между Б и конфетой. Сама конфета не сдвинулась. Сдвинулся мир вокруг неё.
186 424134
>>4132
>>4133
Ну в целом некая абстракция мне понятна, осталось разобраться, что такое движение, расстояние, пространство, ну и время.
63.gif25 Кб, 500x392
187 424135
И гифка твоя говно, в ней корабль просто летит в пространстве быстрее верхнего.
188 424140
>>4132
В данном посте оппонент утверждает, что сжатую область пространства корабль проскочит быстрее, тк:

>Сжимание пространства даёт результат в виде уменьшения расстояния между точками


Похуй, что что генератор, в таком случае придется расположить вне корабля. Да и сам корабль сожмется, вместе с пространством.

И выше, не уверен правда, что тобой, утверждалось, что корабль-то и не двигается (летит с нормальной скоростью), "двигается" пузырь в пространстве. Тоесть премещая "пузырь", ты по сути остаешься в той же точке пространства, а что бы из пузыря выбраться, придется "распрямить" пространство обратно, и в итоге нихуя не произойдет. Закон сохранения энергии.
1.png14 Кб, 591x495
189 424141
>>4140
+
Что бы понятнее: при прохождении "сжатой" области, корабль будет выпрямлять её перед собой (вокруг себя, что ясно из условий), в итоге придется пройти все те же квинтилион километров.

Единственный вариант, это если корабль находится "вне" пространства, но это уже другой корабль.
019.jpeg41 Кб, 400x346
190 424142
Источник тока, это ведь по сути такой хитровыебанный источник напряжения, который подстраивает напряжение так, чтобы ток был заданным, в зависимости от подключенной нагрузки/сопротивления, или нет?

Я просто понять не могу, как может быть ток без напряжения, и для меня напряжение это всегда первопричина тока.
191 424147
>>4140

> а что бы из пузыря выбраться, придется "распрямить" пространство обратно


Вот тут твоя ошибка, пузырь сжимает пространство навсегда, после выключения генератора ничего никуда не разожмётся, там не остаётся такого понятия как сжатое пространство. Если бы генератор только сжимал пространство спереди и не растягивал сзади, то он бы по сути притянул к себе целевую звезду, навсегда сократив расстояние до неё, конечно это нарушило бы кучу законов, поэтому он и растягивает пространство позади, чтобы этого не происходило. То есть результатом его работы становится как бы перемещение куска пространства вместе с кораблём в другое место. Другими словами пузырь как бы пожирает пространство спереди и высерает его сзади.
Дополнительно проясню один момент: при включении генератора он не сжимает сразу кусок пространства и оставляет его в таком состоянии, пока включён, он постоянно поджимает пространство всё сильнее и сильнее, пока его не выключат.
Кстати, расчёты показали, что для работы пузыря в его изначальном виде нужно больше энергии, чем есть во вселенной, и была разработана более совершенная модель, называемая трубой Красникова, для которой и нужны 700 кг антиматерии. Так что если хочешь большей приближенности к практике, то нужно рассматривать эту трубу.
192 424148
>>4142
Такие источники тока, как батарейка или розетка нихуя не подстраивают, они выдают всегда одинаковое напряжение, а сила тока меняется в зависимости от подключённого прибора, то есть да, правильнее было бы называть их источниками напряжения, но привыкли называть их источниками тока. Может бывают и настоящие источники тока, но обычно для них нет задач, проще подстраивать приборы под заданное напряжение, чем наоборот.
193 424149
>>4135
Потому что ты сторонний наблюдатель.
1yMkA51bnrw.jpg86 Кб, 958x538
194 424179
Объясните что такое душа с точки зрения информатики? Если не в западло покидайте видосов на эту тему или литературы
195 424180
>>4179
В информатике такое не рассматривается.
196 424181
>>4180
А как же нейронные сети?
Это разве не некое подобие души?
197 424183
>>4181
Что такое душа, по-твоему?
198 424184
>>4181
Это некое подобие мозга.
199 424200
>>4179
С научной точки зрения, души нет. Вся твоя личность в твоей голове. Крысам научились менять воспоминания, научились с помощью мрт считывать что человек видит перед собой.
200 424207
>>4200
Неужели человека всё же делают только его воспоминания?
201 424213
Это правда что организм блокирует рост клеток чтобы предотвратить рак?
202 424214
>>4207
Если нет, то что? И понадобятся пруфы. Пока что всё указывает на это.
203 424215
>>4214

>Если нет, то что? И понадобятся пруфы.


Это тебе понадобятся пруфы, что ТОЛЬКО воспоминания играют роль. Коих у тебя нет, потому что ты запизделся.
204 424216
>>4215
Хех... Конечно не только воспоминания, еще и физиологическое строение. Но это собственно всё. Еще можно окружение добавить. Но об этом и речь, про взаимодействие с окружением. Если душа есть, то она материальна. Если она нематериальна, то не может влиять никак на нас, ведь всё что может как-то взаимодействовать материально по определению.
205 424219
>>3747 (OP)
Добрый вечер, наукач.
Аспирант 1 курса. Возник вопрос по решению задачи Стефана о фазовом переходе. Ввиду того, что инженер, а нихуя не физмат есть определенные сложности с математикой (тяжко вкуривать в эти ваши диффуры и интегралы).

Буду признателен, если кто-нибудь сможет помочь и пояснить популярным языком какая буковка\циферка за что отвечает. Заранее всем спасибо.
 .jpg350 Кб, 1400x1107
Ваши действия? 206 424226
Вас посетил газофазный ураниевый реактор с МГД генератором и лазерным переносом тепловой энергии в рабочую зоны для нагрева плазмы. Но существующие материалы испаряются при взаимодействии с плазмой. Ваши действия?
207 424229
>>4226
Шизик отъебись.
208 424230
>>4213

>Это правда что организм блокирует рост клеток чтобы предотвратить рак?


Да, есть такая гипотеза, но несмотря на категоричное утверждение русскоязычной Википедии
https://ru.wikipedia.org/wiki/Предел_Хейфлика

>В настоящее время главенствует точка зрения, связывающая лимит Хейфлика с проявлением механизма подавления опухолеобразования, возникшего у многоклеточных организмов.


- все не так однозначно. Главный вопрос, а собственно почему тогда рак, часто, вполне успевает убивать организм?
209 424231
>>4230
Большинство видов рака мутирует так, что у них активируется теломераза и им становится насрать на предел Хейфлика.
210 424232
>>4231
Спосибо КО!
Вопрос как раз в том, какая тогда логика тогда в "объяснении" укорочения теломер - как защиты от рака.
211 424233
>>4232
С укорочением раку нужна +1 мутация, чтобы стать бессмертным, тем самым понижается вероятность развития рака.
212 424234
>>4233
А что, бывает НЕ "бессмертный" рак?
Или наоборот, ты приведешь примеры заболеваний от которых такой механизм помогает (примеры заболеваний с делением клеток - которое благополучно заканчиваются самоизлечением, по причине исчерпания в этих клетках теломер)?
213 424235
>>4234

>А что, бывает НЕ "бессмертный" рак?


Бывает, когда раку не везёт получить такую мутацию, он как раз таки самоизлечивается. Такое протекает совершенно незаметно, до образования больших опухолей.
Однако бывают редкие виды рака, с неактивной теломеразой, которые почему-то не умирают от предела Хейфлика, такие виды ещё малоизучены.
214 424236
>>4235

>Бывает, когда раку не везёт получить такую мутацию, он как раз таки самоизлечивается.


А ты перенесешь пруфы , что самоизлечение подобного связанно именно с пределом Хейфлика? Или хотя бы на то что подобное называют "рак" (мне всегда казалось, что злокачественная/доброкачественная опухоль - отличается отнюдь не по мутации теломеразы)?
215 424237
>>4236
Я не говорил, что это доброкачественная опухоль. Может это и нельзя считать раком, это не важно, это просто один из этапов развития рака, который может закончиться двумя путями.
216 424240
Почему школьные учебники по физике говорят что замкнутая цепь - обязательное условие существования электрического тока(в ней). Ведь молния пробивает просто от полюса к полюсу и баланс тем самым восстановлен. Молния это же тоже ток, да ? Что будет если 2 одинаковые батарейки соединить + к -, а другие концы не трогать ?
217 424242
>>4240
Помоему нихуя не будеь
Ну или батареи разрядяться
218 424243
>>4240
При особо мощном напряжении проводником считается абсолютно все.

Да ничего не будет, слишком много электронов/дырок на свободных концах появится но это просто повысит общее напряжение (по формуле, ага)
219 424244
>>4242
>>4243
Но ведь если я конденсатор заряжу, а потом подключу замкну его контакты проводом, то очень быстро все электроны распределятся равномерно, так ?
220 424245
>>4240

>электрического тока


Кхм... постоянного, да. Если переменного, то не обязательно.
221 424246
>>4230
Как тогда такая аналогия с реальным миром: Совок сам себя убил. Нагрузка на логистику. Потому что не ограничил рост. И рост этот был не качественным. И собственно вопрос тогда: китай сам себя убьет?
222 424247
>>4245
Вот честно говоря я подзабыл. В конденсаторе ток есть, если он в цепи переменного тока. Но это не ток проводимости, а ток смещения, который и ввели ради описания данного явления. Если его учитывать, то цепь замкнута. Надо бы почитать пролетают там заряды то или нет в деэлектрике конденсатора.
223 424249
>>4237
Я так понял ссылок не будет... но все равно - спасибо (хотя, подозреваю, что ты ни разу не специалист в онкологии).
224 424251
>>4246
При рассуждении по аналогии - ответственность за корректность аналогии - на рассуждающем.
(На самом деле твое рассуждение:
[развитие государства]~[развитие организма]
- это даже на аналогию не тянет, в лучшем случаи метафора/притча, в худшем - т.н. "магическое мышление").
225 424252
>>4249
Разрабатываю методы ранней диагностики рака.
226 424253
Помогите, нужно сделать проект в шарагу, электрогитару самодельную, как максимально дёшево сделать звукосниматель? Гитару абсолютно любую, главное что бы принцип был гитары
227 424254
>>3747 (OP)
Как отличаются по постановке задачи и по проведению доказательств доказательства "тогда" и "только тогда"?
228 424271
Не уверен, что по адресу, но спрошу:
есть какая- нибудь годная литература по философии?
229 424279
>>4271
Если хочешь быть подкаблучником - Гегель.
Если нет - то Шопенгауэр. Только найди что-то с самым скучным названием.
230 424282
Почему еще не создали ионную/протонную/нейтронную/гаусс/плазмменную пушку?
231 424283
>>4282
+лазган
232 424287
>>4282
>>4283
Дорого и не нужно
233 424294
>>4279
А что- то вроде университетского курса введения в общую философию?
234 424295
>>4247
Нет, ток смещения это просто изменение E. Четвертое уравнение Максвела.
235 424308
Посоны, внезапно осознал, что нихуя не знаю о красном смещении. Википедия говорит, что есть два явления с таким названием (от чего я мальца прихуел). Ну да не суть. Как люди узнали о существовании сего феномена? Почему все подумали, что частота приходящего к нам излучения смещена в красную область, а то, что мы фиксируем приборами не является естественным спектром светимости небесных тел? Надеюсь смог понятно сформулировать вопрос.
236 424311
>>4308
Спеектрыы газов
237 424312
>>4311
Можно подробнее? Я понимаю, что по спектральным линиям в свете звезд можно определить их химический состав. Но откуда узнали, что линии одного химического элемента, смещенные в красную область в спектре света от далеких звезд - это не линии другого элемента (или изотопа), находящегося на своем месте?
238 424316
>>4312
Линий много, и у каждого газа целый набор со своими интенсивностями стандартными(почти).
239 424320
>>4271

>Не уверен, что по адресу


Не по адресу, тебе в /ph/
240 424321
>>4282

>ионную


Создали ионный космический двигатель, в этом направлении применение ионов гораздо полезнее, чем в оружии.

>протонную


Протоны будут отталкиваться друг от друга, рассеивая пучок.

>нейтронную


У нейтронов слишком короткое время жизни.

>гаусс


Устаревшее говно без задач, рельса лучше.

>плазмменную


До сих пор не найдено способа придания стабильности плазме.
>>4283
Слишком большие потери энергии в воздухе, в космосе ещё имело бы смысл.
241 424322
>>4308
Ну смотри, если объект удаляется, то по законам механики, испускаемый им свет должен быть медленнее на ту скорость, с которой удаляется объект. Но свет не может быть медленнее или быстрее, поэтому вместо изменения скорости он изменяет частоту.
242 424323
>>4312

>Но откуда узнали, что линии одного химического элемента, смещенные в красную область в спектре света от далеких звезд - это не линии другого элемента


На земле/солнце - элементов с таким спектром не наблюдается, а вот если сдвинуть линии в их спектрах с учетом красного смещения, то как раз получаются вполне земные элементы. Это если упрощенно.
На самом деле смещение частоты при движение - это банальность, во вполне а земных условиях наблюдается/используется, доплеровские радары - вот это все. А эталон частоты приходится охлаждать, что бы тепловое движение помехи не вносило.
243 424325
>>4282
На wm спроси. Только будь готов что тебя там обоссут, выебут мамку, и насуют за щеку.
244 424327
>>4325
Ну это в любом разделе сделать могут.
245 424329
Вижу сотни статей каждый месяц, сам Хокинг писал, что не знает, что происходит с "информацией" в черной дыре.
Но ведь все сошлись на принипе схлопывании, типа ядерный взрыв наоборот, с уничтожением энергии, и то что главный эффект дыры - это то что лучи света и всё остальное не имеет достаточной силы чтобы выбраться из этой засасывающей дыры.
Эти принципы же настолько очевидны, что может понять любой школьник. Так зачем пишут что типа время растекается происходят аномалии и кротовые дыры, если просто блядь всё аннигилируется? Это как совершенный пресс - попадет любое тело, планета в эту воронку и тупо спрессуется, уничтожится. О какой сука информации они спорят, почему все такие долбоебы?
246 424336
>>4329
Почему ты такой долбоёб, что даже не знаешь об "испарении" чёрных дыр за счёт излучения Хокинга?
247 424339
>>4329
Ты просто настолько далек от понимания и представления, что даже не понимаешь всей проблемы. Это как плоскоземельщик будет удивляться проблемам с приливами и навигацией самолетов. "Они что дебилы там?"
248 424340
>>4329
Не слушай даунов выше. Проблема уничтожения информации просто тобой не правильно понята.
Вот представь, что ты повар. И готовишь ты хлеб. В зависимости от того, что ты кинешь в тесто, готовый хлеб будет иметь разный вкус: кинешь много соли - будет солёный, кинешь много сахара - будет сладкий. Смекаешь? Вот и так в реальном мире. Все действия имеют последствия и, если пошевелить мозгами, то из конечного варианта можно узнать, что было в начальном варианте. Вот тут все и начинают охуевать с черных дыр: что бы ты не бросил в тесто, хоть ты говна туда наложи, хоть меда и икры черной - на выходе все равно будет всегда абсолютно одинаковый продукт.

Вот в этом и проблема. ЧД единственная кто игнорирует что в неё попало. А законы говорят, что такого быть не может.
249 424341
>>4336
Испарение происходит с поверхности. Из-за горизонта не излучается ничего.
250 424342
>>4340

>А законы говорят, что такого быть не может


Кокие?
251 424343
>>4342
Например, заряды элементарных частиц уничтожаются.
252 424344
>>4341
Да, но чёрная дыра уменьшается.
>>4343
Нет, ведь бывают заряженные чёрные дыры.
15025769452791.jpg113 Кб, 1273x922
253 424345
>>4344

>ведь бывают заряженные чёрные дыры

254 424346
Как рассчитать количество возможных квантовых состояний в каком-либо теле? реквестирую формулу
255 424348
>>4343
>>4345

>Например, заряды элементарных частиц уничтожаются


Чего только не прочитаешь у экспертов Двача. И главное так уверенно пишут, за потерю информации в ЧД поясняют, с помощью прямо таки библейского калибра притч
>>4340
Вот представь, что ты повар. И готовишь ты хлеб. В зависимости от того, что ты кинешь в тесто, готовый хлеб будет иметь разный вкус: кинешь много соли - будет солёный, кинешь много сахара - будет сладкий. Смекаешь? Вот и так в реальном мире. Все действия имеют последствия и, если пошевелить мозгами, то из конечного варианта можно узнать, что было в начальном варианте. Вот тут все и начинают охуевать с черных дыр: что бы ты не бросил в тесто, хоть ты говна туда наложи, хоть меда и икры черной - на выходе все равно будет всегда абсолютно одинаковый продукт.
Евангелие от Анона, Наук. 423747:424329


https://www.youtube.com/watch?v=uB4zwhvaKLE
256 424351
>>4348

>пуук


Че по делу то скажешь?)))
257 424353
>>4340
Они просто рассматривают дыры, как абсолютно плоские поверхности. А что если по горизонту событий бегают постоянно волны. И по этим волнам, можно понять из чего она.
258 424354
>>4346
Их бесконечно). Или тебе плотность их на какой-то конкретной энергии нужна?
259 424357
>>4353

>И по этим волнам, можно понять из чего она


Для стороннего наблюдателя - никак нельзя. Ведь дыра просто в себя впитывает все без разбора и изменяет только свою массу.
260 424358
>>4351
>>4357

>Ведь дыра просто в себя впитывает все без разбора и изменяет только свою массу.


>изменяет только свою массу


С вас, дегенератов, пруфы, что ЧД, кроме массы, не меняет еще заряд и момент импульса (если упавшее тело было заряжено и двигалось соответствующим образом).
261 424359
>>4358

>заряд


Какой заряд? Или ты хочешь сказать, что можно вот так вот подойти, опустить термометр в дыру и вынуть обратно?
262 424360
>>4358
Меня то ты зачем туда записал. Я про заряд ничего не говорил. Вполне себе сохраняют черные дыры заряд. Они даже магнитное поле имеют некоторые. Я про то, что возможно, если на повержности дыры существуют волны горизонта событий, то излучение Хокинга не имеет сферическую симметрию, и оно уносит инфу обратно. Тогда никакой термодинамической проблемы нет.
263 424361
>>3747 (OP)
Сап, я студент меда тупой пекус, перевожу статью по молекулярной биологии. Так как я делаю это первый раз, мне нужен человек, который уже более-менее сведущ в данном вопросе (проблемы вызывают не сами слова, а тонкости перевода фраз), кому интересно помочь мне в этом начинании, оставьте контакты. Заранее извиняюсь, если я не в тот тред написал
264 424362
>>4359

>заряд


>термометр


Смешно - надеюсь что это шутка была. А температуру ЧДе - вполне можно приписать, как температуру АЧТ соответствующего данному излучения Хокинга.

>>4360

>Меня то ты зачем туда записал.


Тогда, мои извинения.
265 424364
>>4362

>Смешно


А еще смешнее будет услышать, как ты собрался измерять заряд ЧД
M4PwtG20hbI.jpg100 Кб, 1080x648
266 424367
Может тут кто знает что за язык
268 424372

>Гибриды между генетически различными линиями сильнее и жизнеспособнее, чем родители. Это используется в основном для получения гибридов F1 в растениеводстве, у млекопитающих выражено слабее, но тем не менее выражено. Метисы между людьми разных рас обычно бывают здоровее и умнее, чем потомки многовекового инбридинга. В качестве потомка двух представителей разных видов, а не рас, можно взять мула. Он сильнее, выносливее, неприхотливее лошадей и ослов, живет гораздо дольше и тех, и других.



Если возьмем белую сучку и нигера с огромным хуем и скрестим их, то их дети будут сильнее и умнее, с теми, кто родился от расово одинаковых людей?
А вот если их детей скрестить с азиатами, то они будут еще сильнее и умнее? Так можно делать до бесконечности?
269 424374
>>4372
Можно ли сделать супер людей таким образом?
1397234739304.jpg33 Кб, 403x336
270 424394
Диаграмма агрегатного состояния золота.
Запостите пожалуйста. Спасибо.
271 424396
Аноны, сориентируйте по воскрешению знаний по математике
В школе умел в неё норм, на олимпиадки даж ездил не сильно далеко. Но потом лоханулся с универом, теперь спустя почти 10 лет после школы чувствую себя пустым без математики родненькой в голове.

Собственно практической цели нет, хочется просто освежить школьный курс и математическую статистику с теорией вероятности. Мб книги конкретные засоветуете, или где-нить схемы есть в каком порядке проходить?
autism.png277 Кб, 1470x992
272 424400
Какова собственная скорость движения галактик, относительно гипотетического центра Вселенной после Большого Взрыва?
Т.е. какая скорость была получена галактиками вследствие Большого Взрыва.
273 424401
>>4400
Никакой. Врыв это не из точки. Это лишь название такое. Это не буквально.
274 424403
>>4400
Взрыв этот является 4 мерным. Где центр от нас находится в 13 млрд. лет.
Ввиду этого скорость этого взрыва 1 секунда за одну секунду.
Чему равна 1 секунда в метрах неизвестно. Но можно предположить что 300 000 км.
Итого предпологаемая скорость эта равна - с.
Но это фентези так что ответа нет.
275 424407
>>4400
Зависит от расстояния, ближние галактики движутся медленнее, дальние - быстрее.
276 424409
>>4403
бля, что ты несешь:

>Взрыв этот является 4 мерным.


точка в любом измерении это, блядь, точка.

>Ввиду этого скорость этого взрыва 1 секунда за одну секунду.


и плотность 1 килограмм на 1 килограмм, ога.

>Чему равна 1 секунда в метрах неизвестно.


как и мне.

>Итого предпологаемая скорость эта равна - с


так ты имел ввиду световую секунду?
277 424410
>>4409
Нет я имел ввиду дистанцию на оси времени. И эта твоя точка 4 мерного объекта будет только в одном 3 мерном срезе если резать параллельно, конечно. Говоря простыми словами, центр вселенной находится в прошлом ну или в будущем, хуй знает..
278 424428
>>4367
>>4370
некоторые знаки похожи на тибетские, бенгальские, монгольские, но гораздо больше всё это похоже на фантазию художника
279 424429
Проскочила инфа, что американцы очень активно собирают ген материалы россиян. Зачем? Для чего? Какое стратегическое преимущество обладание ген базой противника может дать в реальных боевых действиях? Можно создать биооружие которое будет атаковать только русских?
autism.png277 Кб, 1470x992
280 424432
Я сам не так давно читал, что есть собственная скорость движения галактик. Она небольшая и, вероятно, сотни километров в секунду, точно не помню.
281 424435
>>4432
Да, есть. Они стягиваются в свои скопления, но в среднем должно получиться близко к нулю. Ибо тогда появляется выделенный центр, и нарушается симметрия
282 424436
>>4429
Проскочила инфа, что ты школьную программу по генетике не осилил.
283 424438
Анон скажи, что за пылевые узоры на крышке? Статическое электричество? Во время ремонта крышки от распределительных коробок валялись несколько дней в разных местах, на всех такие узоры от пыли
284 424439
>>4436
В любом случае вопрос этот задал сам президент, а у него наверняка куча био-советников, так что вопрос релевантный и открытый для обсуждения. Если нечего сказать по существу то можешь начать ДЗ делать.
285 424442
>>4429
>>4439

>вопрос релевантный и открытый для обсуждения


Да, и теперь вопрос из области психологии детства - как из этого подтвержденного факта:

>американцы ... собирают ген материалы россиян


- ты перескочил к этой гипотезе:

>преимущество обладание ген базой противника может дать в реальных боевых действиях


?
(Впрочем, эта гипотеза сформулирована в форме вопроса, а не категоричного утверждения, что уже хорошо.)

Намек:

>стратегическое преимущество


- это, в первую очередь, - экономика вообще и экономическое прогнозирование в частности.
286 424444
>>4442
Ты не прошёл тест Тьюринга. с трудом верится что это мог написать не чат бот.
287 424447
>>4429

> Зачем? Для чего?


Они говорят, что разрабатывают какое-то лекарство, причём уже давно, и раньше они испытывали его на русских, поэтому и продолжают работать с ними. Почему же они раньше начали с русских? Хуй знает, у них надо спрашивать.

> Какое стратегическое преимущество обладание ген базой противника может дать в реальных боевых действиях?


Никакого, среднестатистическая разница в геноме представителей разных рас и национальностей ничтожно мала.

> Можно создать биооружие которое будет атаковать только русских?


Нельзя, причину см. выше.
288 424460
>>3747 (OP)
Ученый-анон, что представляет из себя информация, например на жестком диске, в материальном виде. Оно же как-то там хранится. Это какие-то частицы может быть, или импульсы там электрические, не знаю крч я неуч, поясните плес.
289 424461
>>4460
Там короч микроскопические кусочки металла намагничиваются.
290 424462
>>4460
Информация вообще не материальна.
291 424464
>>4435
нет, речь про собственную скорость относительно вселенной, а нем соседних объектов
292 424465
>>4464

>собственную скорость относительно вселенной


Простите... это как?
293 424468
>>4435
в другом месте выёбывайся, мудак
можешь в википедию ещё сходить или на физфорумы, там полно таких дегенератов
294 424470
>>4468
??? спонтанный взрыв на пустом месте?
295 424473
>>4438
Трещины, же.
296 424482
Время абстрактно?
297 424484
>>4482
Всё абстрактно
298 424489
>>4482
Учитывая что твоя комната, это лишь воображаемая модель в твоей голове, то всё абстрактно.
299 424491
>>4482
нет
300 424492
Почему говорят, что для значимых изменений вида нужны тысячи поколений эволюции, но при этом за 200 лет люди на 20см выше стали? и правда ли это?
301 424493
>>4492

>но при этом за 200 лет люди на 20см выше стали?


Не думай о всякой хуйне, лучше ещё курочки с гормонами роста наверни.
302 424495
>>4492

>для значимых изменений вида нужны тысячи поколений эволюции.


>за 200 лет люди на 20см выше стали


Сделай вывод.
303 424496
>>4493
Так я и не думаю, я спрашиваю. Думать вам.
>>4495
Не могу. Нельзя же просто ёбнуть гормона роста и стать выше, при этом чтоб всё нормально работало. Так бы все карлики лечились. Нужна тонкая подстройка всей эндокринной системы генами. Хуй знает, в общем, мой разум в тупике. Разве что из-за плохого питания, но тогда всякие графья и купцы должны были быть выше, чего вроде не было.
304 424497
"Трансформеры"
Зерно (англ. Seed) - артефакт, превращающий органические материалы в трансформий.

Вопрос: Реально ли это и может ли быть реально в будущем? Превращать органику в какой либо металл?
Очень крутая технология бы вышла, я считаю!
305 424498
Передаётся ли звук в сверхтекучей жидкости без вязкой компоненты? Будет ли она оказывать сопротивление движущемуся в ней телу - пуле, автомобилю?
306 424501
>>4465
это для тебя, пидор, хз как другому челу отправилось, пардон
>>4468
307 424502
>>4501
Ты понимаешь, что я даже вопроса не понял. Как ты можешь измерять скорость относительно ничего. Относительно какой точки вселенной ты измерять хочешь?
308 424532
>>4502
Относительно реликтового фона.
мимо
309 424543
>>4496

>Нужна тонкая подстройка всей эндокринной системы генами


Берешь 2 группы бычков, желательно от одной породы. Первую группу кормишь хорошо, а второю лишь бы не подохли. Через год измеряешь всех бычков и о чудо - первая группа состоит из куда более крупных и сильных животных! Вот она, сила эволюции! Всего за год изменила целую группу животных, а не мильон лет, как написано в книжке!
310 424544
>>4532
что мимо... что мимо... ты понимаешь, что вопрос невозможно понять, если человек не опишет как он себе это представил. Представь я буду спрашивать у тебя вопросы про автомобили, представляя мотоцикл с коляской.
-А где у автомобиля коляска? Справа или слева?
-Не понял вопроса, у машин нет коляски, может ты про прицеп? О чем ты?
- Относительно переднего колеса. мимо
311 424553
>>4544
Я полагаю, что он говорил о скорости движения местной группы галактик относительно дипольной анизотропии реликтового излучения, которая составляет ~627 км/с.
312 424555
>>4553
Ну это будет как ответ про прицеп. Вообще многих безумно путает название "большой взрыв". Людям кажется, будто есть конкретыный центр, от которого всё разлетается.
313 424580
Что дает вычисление протеинов?
1136713.jpg39 Кб, 586x700
Лев Гумилев и культурология 314 424594
Какой вклад Лев Гумилев внес в культурологию? Как вообще можно связать этого ученого и культурологию в общем контексте??
315 424596
Парни, как понять проблему горизонта? Есть решение? Я не про чёрные дыры.
316 424600
>>4596
Как то нелепо это слушать от нефтедержавы
317 424618
>>4580
Бамп
318 424620
Анон, подскажи примеры фенотипов у людей. Спасибо.
1482757426272-0.jpg22 Кб, 267x274
319 424638
1. По какому методу определить скорость теплопередачи в твёрдом теле? Вот, допустим, есть тело длиной 1м, один торец греют на +100К. За какое время температура с горячего торца полностью дойдёт до холодного?

2. По какому методу определить скорость конвекции? Т.е. то же самое, но применительно к жидкости/газу.
320 424639
>>4638

>За какое время температура с горячего торца полностью дойдёт до холодного?


>температура ... дойдёт до холодного


Ты понимаешь, что даже с т.з. бытового здравого смысла (не говоря про физику) довольно странная?
321 424640
>>4639

>не говоря про физику) довольно странная?


не говоря про физику) эта твоя фраза - довольно странная?
322 424641
>>4638
Неграмотно с точки зрения физики написано, но вроде понял, что надо.
1. Основной закон теплопроводности Фурье.
2. Он же, просто коэффициент поменяется.
323 424643
>>4639

>Ты понимаешь, что даже с т.з. бытового здравого смысла (не говоря про физику) довольно странная?


Чего странного? Вот пытаю я человека паяльником. Паяльник имеет жало определённой длины, которое одним концом воткнуто в анус, а на другом конце закреплён нагревательный элемент. (Для простоты считаем, что он разогрелся мгновенно и получает дополнительную энергию из розетки.) Необходимо найти время, через которое анус будет жечь максимальной температурой.

>>4641
Вот с конвекцией непонятно нихуя. Там ведь помимо теплопередачи ещё и тепловое расширение добавляется, которое нелинейно увеличивает скорость переноса тепла.
324 424644
>>4620
Горб, сутулость.
325 424647
Анон, есть закупоренная стеклянная бутылка с водно-спиртовым 40% раствором (водкой) , пролежавшая в Балтийском море 100 лет. На что надеяться, если откупорить эту бутылку. Спирт сохранился? Или заменился морской водой?
326 424648
>>4647

>На что надеяться, если откупорить эту бутылку.


Ватный джинн вылетит.
327 424651
>>4647
Да что с ней могло случиться то. Если пробка хорошая, то там водка.
328 424652
>>4596
Бамп
329 424656
>>4643
Основной смысл закона Фурье в том, что теплопроводность пропорциональна градиенту температуры. Это верно для тел в любом агрегатном состоянии. Все остальные детали заключены в коэффициенте. Для газов есть приближённые теоретические формулы вычисления коэффициента, но лучшие результаты даёт опыт, а опытные данные можно найти в банальных таблицах.
330 424657
Как называется веревка для суицида и как она может пригодиться в хозяйстве?
331 424662
>>4462
пока дело не дошло до термодинамики и горизонтов событий.
332 424671
>>4651
Когда я нашёл в 2014 водку 1993 года в ней было две трети жидкости крепостью градусов 30
333 424672
>>4671
значит или водка хуевая или пробка. Спирт не может вот так просто исчезнуть вникуда.
334 424676
>>4672
Спирт вступает в реакцию выщелачивания стекла. Со временем стекло превращается в полупроницаемую мебрану и начинает пропускать воду - физическое явление осмоса. Объем жидкости внутри бутылки, хранимой под водой, останется неизменным или увеличится. А вот, если хранить бутылку в сухом темном месте, то после реакции выщелачивания стекла объем жидкости уменьшится, появится осадок (не всегда) и изменится вкус напитка. В ГОСТе устанавливается срок хранения крепких спиртных напитков в стекле, но не указывается срок годности.
http://vlr53.narod.ru/izmeneniya.htm
https://forum.guns.ru/forummessage/21/000258.html
http://www.vladtime.ru/allworld/392700
335 424678
>>4676
получается спирт и жидкость испаряются с поверхности бутылки? Проходить может только вода? А, скажем, ионы лития могут? Неплохой бы ионистор/аккумулятор бы получился.
336 424679
Чему будет равны выражения
бесконечность - (бесконечность+1)=
бесконечность - (бесконечность-1)=
337 424680
>>4679
#define ∞ "бесконечность"
int a=∞;
a-(a+1)=a-a-1=-1;
a-(a-1)=a-a+1=+1;

пускай бесконечность и не является числом, выражения вида"∞-∞" смело можно приравнивать к нулю т.к. это "одинаковые" бесконечности. А вот "∞+∞2" уже нет.
изображение.png51 Кб, 1686x453
338 424681
>>4679
В матанализе результат может быть различным
>>4680
У тебя а - обычное число, а не бесконечность
339 424682
>>4681
бля ну написал же комментарий в конце чтоб уточнить что не каждые действия вот так можно проводить. Так вообщет пределы решаются - только наоборот, вместо переменной ∞ подставляется и смотрится что на выходе, если не понятно, продолжаем прощать.
340 424686
>>4678
Специальное электродное стекло надо. Старые pH-метры на этом основаны http://mirznanii.com/a/324438/steklyannye-elektrody-i-ikh-priminenie
Подозреваю, что сохранность водки на морском дне сильно зависит от состава стекла. Примесь лития в стекле могла ускорить осмотические процессы.
341 424689
>>4012
Отвечаю: мультиметром измерить объем нельзя, только определить до какого напряжения зарядился от зарядника, а имея эталонные значения оценить примерную текущую емкость. А нагрузочная вилка оценивает состояние аккума по просадке напряжения под нагрузкой, если слишком сильно падает, значит часть ячеек уже не работает, общая емкость будет меньше изначальной, ну и таблицы с примерным соответствием вольтажа и емкости. Убитый аккум может зарядится до норм напряжения, но сдохнуть под нагрузкой за 5 минут.
Для нормального измерения вообще надо батарею разрядить на электронной нагрузке, которая посчитает отданную емкость, при этом значение будет разное в зависимости от тока разряда и условий теста.
342 424695
>>4679
Бесконечности бывают разные)
343 424698
>>4322
>>4316
>>4323
Запоздало благодарю за ответы.

Собственно еще меня волнует следующий вопрос. Вселенная расширяется, причем расширяется одновременно в каждой своей точке (пздц сколько аналогий с воздушными шариками и резиновыми скатертями по этому поводу в инете, видимо, очень популярная тема). Излучение от далеких звезд и прочих излучателей до нас идет миллиарды световых лет. Почему за это время расширение пространства не растягивает фотоны в лучах, и мы не видим эти объекты "пунктирами"?
344 424699
>>4698
Пунктирами и видим. Самые дальние звездочки именно что и делают "мигают".
345 424703
Для беспородных кошек и собак мука и мучные изделия являются ядом или вполне нормальной едой ? А для диких вариантов типа тигров и волков ? Т. е. они же все хищники, значит чрезмерное потребление некоторых продуктов растительного происхождения для них вредно ?
346 424714
Как снять статическое электричество с волос?
Отрастил патлы, купил фен, настал пиздец. Куда скидывать заряд? Ебанитовая расчёска поможет или нет?
347 424719
как преодолеть горизонт событий?
теоритически это возможно?
348 424722
>>4714
к батарее приложись
349 424723
>>3747 (OP)
Сводится ли гипотетическое утверждение к категорическому?
350 424729
>>4719
Возможно залетай в ГС и обмазывайся сингулярностями.
351 424730
А вот такой вопрос. Вот падает вещество на ГС. Оно получается телепортируеться в центр ЧД?
352 424731
>>3747 (OP)
Чем отличается аксиома выбора и аксиома детерминированности?
Что бы я имел общее точное представление и на время успокоился.
353 424732
>>4722
Она окрашена.
354 424736
Можно глюоны разогнать до скорости света?
355 424737
>>4732
вот когда сосед заземлит на эту батарею водонагреватель или еще какую хуйню, тебя может ебнуть вне зависимости от наличия краски
356 424738
Почему электрон, вращаясь вокруг ядра должен излучать?
357 424739
>>4737
А статика с волос при этом сбросится?
358 424740
Хочу немного подифференциировать и поинтегрировать для практики. Посоветуйте небольшой задачник на тему, где все проблемы будут поставлены не в духе "посчитать интеграл (a + x)^2/(sin(x) + (x + b)^3 dx", а как "посчитать среднюю высоту графика такой-то функции", "найти площадь", решить какую-нибудь достаточно простую и очевидно полезную физическую/геометрическую задачку.
359 424741
>>4738
Потому что у него ненулевой заряд. Ненулевой заряд при движении создаёт поле.
360 424743
>>4714

>Ебанитовая расчёска поможет или нет?


Наоборот же, нужна простая металлическая.

>>4732

>Она окрашена.


Можно за кран.
361 424744
>>4743
По поводу 2-го пика - уточню, что "беспроводной антистатический браслет" - это какая то ересь, а вот с поводком-проводом - вполне на производстве применяют, сам видел.
362 424745
>>4741
Ну создал он поле, почему там фотон должен вылететь?
363 424746
>>4743

>спрей антистатик


Как эта хуйня работает? Она же заряд не убирает.

>2 пик


Можно же просто подержать в руке связку ключей и заземлить о батарею.
364 424758
>>4745
Поле из фотонов состоит
365 424762
>>4758
Ну ок, состоит, но почему при вращении электрона они должны излучаться?
И по-моему ты уже забегаешь в кваны, а мне интересно как пришли к тому что планетарная модель атома хуйня негодна.
366 424763
>>4762

>кванты

367 424764
>>3751
Можно ли высрать кишку если сильно тужится?
368 424775
Что находится под пространством?
369 424777
>>4775
Подпространство
370 424779
>>4775
Там ничего нет
371 424780
Интересуют матан и программирование. Еще, правда, биология, особенно все что связано с деятельностью мозга, однако ее я
нихуя не понимаю и никогда не понимал.
В какую сферу науки вкатываться? Думаю между биоинформатикой/биоинженерией, защитой данных и чистой компуктер скинс.

Мимо выкатываюсь из айти программист по образованию
372 424786
>>4780
offtop mode on

>Мимо выкатываюсь из айти программист по образованию


Какая максимальная зарплата была, если не секрет?
offtop mode off
373 424787
Не гоните сразу в /po/.

Учась в вузике, услышал от препода по экономике с вялотекущим совком головного мозга высказывание "Конечная стадия развития капитализма - рабовладельческий строй". Смутно помню, его выводы были связаны с автоматизацией и стоимостью труда (тип расходы на наемного работника в конечном счете станут выше, чем на роботов и автоматы, стоимость труда упадет и люди будут готовы давать в жопу работать за еду).

Таки есть ли научные (не наивно-совковые) доводы на эту тему?

Сам к экономике отношения не имею. Гуглить пробовал - не нашел.
374 424788
>>4786
40к в моей мухосрани, устроился еще студентом на половину, пока доучивался понял что кодинг - дно, не имеющее отношение к моим романтическим представлениям о задротстве матана и алгоритмов; а попасть куда-то, где делают что-либо реально полезное для общества а не очередную подушку-перделку на айфон почти нереально.
Вот и думаю выкатываться, едва вкатившись на полную ставку и поимев средства на выживание и получение допобразования.
375 424790
>>4788

>попасть куда-то, где делают что-либо реально полезное для общества а не очередную подушку-перделку на айфон почти нереально.


Как вариант можно помогать пилить какой-нибудь опенсорс проект вселенской важности (например проекты с научным уклоном) в свободное от работы время.
А насчет науки, слышал, что с физкека куда по идее поступают ужас как увлеченные наукой абитуриенты после окончания бакалавриата многие кончают экономической магистратурой. Одной наукой и хуйцами с гречкой сыт не будешь.
376 424791
>>4790
Ну то физкек, или ты в церн идешь, или в преподы 18-ого лицея для слабоумных детей индиго, среднего класса нет. А та же биоинформатика, вроде как, востребована.
001.jpg77 Кб, 1024x1024
377 424792
>>3747 (OP)
Есть тут кто? У меня вопрос по задаче. Даны массы двух тел, угол столкновения, их скорости, столкновение пластичное, тела после столкновения двигаются дальше.

Нашёл сначала импульс П3, а затем из него скорость. Но ответ неправильный почему? В чём моя логика не верна?
378 424793
>>4792
Всё верно. Арифметикой может ошибся?
379 424795
>>4787
Читал как-то изыскания на эту тему. Не вспомню, какая-то монография постосовкового времени. Короче, поменяется характер труда. Не будет работы, связанной с тяжелым физическим трудом, будет работа, связанная с разработкой роботов и прочей автоматики. Особо ленивые и тупые будут, например, заниматься псевдо-творчеством: дизайном тех же роботов, промышленной эстетикой, или организационно-финансовыми вопросами, на которые раньше не хватало ресурсов. Можно пособирать выборки из открытых источников взаимосвязь количество роботов и рост заработной платы по стране.
001.jpg35 Кб, 903x180
380 424796
>>4793
Возможно, однако вот формула, по которой это всё считается и меня раздражает, что нужно оказывается оба вектора приводить так сказать к осям от чего сам суммарный вектор естественно увеличивается. Число другое. Впервые в жизни я настолько сильно ошибаюсь в своих мыслях при решении задач по физике.
381 424797
>>4796
И вдогонку, по которой я считал изначально. Может действительно я чего не понял?
383 424799
>>4798
Про корень забыл, но с ним считал.
384 424800
>>4798
Да, ты ошибся. Смотри у тебя по теореме косинусов не тот угол. Ты можешь просто воткнуть одну ось по P1. тогда компоненты будут .
(P1,0); А у второго тогда (cos(a)P2;sin(a)P2); суммарный вектор. (P1+P2cos(a);P2sin(a));
Берем меру. Корень(P1^2+P2^2+2P1P2cos(a));
Тот же результат. Можно получить, если понять, что в твоем решении вместо a в косинус надо посдтавить (180-a); что cos(180-a)=-cos(a);
385 424802
>>4800
Спасибо за разъяснение.
386 424804
>>4792

>пластичное


Не абсолютно упругое? Тела деформировались и после удара двигаются как единое целове? По идее часть энергии должно перейти в ногрев, но я хз, в школах всегда были с этим проблемы, хуй знает какое очередное допущение неявно подразумевалось в задаче, а какие нет.
387 424807
>>3747 (OP)
Вот можно было бы сделать чтобы заниматься однообразной работой и не уставать
388 424809
Что будет, если все вирусы умрут?
389 424810
>>4804

>По идее часть энергии должно перейти в нагрев ...


Все верно, поэтому эта задача не сохранение импульса а не энергии.

(А автор вопроса, как правильно сказали, просто перепутал угол

>у тебя по теореме косинусов не тот угол.


)
390 424811
>>4809
Что будет, если весь водород, углерод, гелий умрёт?
391 424812
Имею высшее техническое образование, с биологией никак не связан, но со школы её люблю. Реквестирую объяснение на пальцах разницы между фенотипом и генотипом. Вижу что это, можно сказать платина, и здесь часто тралят на эту тему. Но сколько бы я не читал единой точки зрения в голове не возникает. Со школы я думал что фенотип определяется генотипом и средой. Недавно услышал С. В. Савельева, что никак форма геномом не определяется. Временами встречаются эпигенетические высказывания о возможности наследования внешних признаков(и поведения и структуры тела). Вот взять гусеницу, которая должна стать бабочкой и поместить её в какие-то особые условия, может быть так что у нас вырастит необычно большая(например) сытая гусеница, которая так и будет жить не превращаясь в бабочку, возможно даже много больше средней продолжительности жизни ?
392 424813
>>4812

>Недавно услышал С. В. Савельева


В этом твоя ошибка, больше никогда не слушай его.
393 424814
>>4813
Без обид, но анон с двача не может вот так просто обвинить учёного из телевизора в неправоте. Если есть какие-то разные школы/течения, то мой вопрос как разв том чтобы мне их кратко описали.
394 424816
>>4814

>учёного


В этом твоя вторая ошибка, он не является учёным.
395 424817
>>4816
Он советский учёный, самый лучший из возможных. Больше таких не делают.
396 424825
>>4817
Перетолстил что ли?
397 424830
>>4825
Угу. Самым жирным даже не отвечают.
398 424873
>>4762
бамп
399 424875
>>4807
Можно, но не долго. амфетамин
400 424876
Как с научной точки зрения объяснить тот факт, что я девственник в 30 лет?
401 424877
>>4876
Родился не в том месте и не в то время
402 424878
>>4877
Это ненаучное объяснение, и вообще не объяснение. Поскольку не объясняет ничего.
403 424879
>>4878
А какое объяснение ты ожидаешь? Может у тебя хуя нет, или тебе просто не нужны тян, откуда нам знать?
404 424885
>>4762

>а мне интересно как пришли к тому что планетарная модель атома хуйня негодна.


посмотри какой-нибудь научпоп про историю квантмеха
405 424886
>>4876
Естественный отбор. Ты не прошел.
406 424891
>>4699
Они из-за атмосферы мерцают же. А в космосе горят непрерывно.
407 424892
>>4876
Опишите свою проблему подробнее.
image.png25 Кб, 450x100
408 424893
>>3747 (OP)
Поясните за формулу Таппера, пожалуйста. Она и в правду работает? Как так получается? Как ее вывели?
Lifeinlife.webm10,5 Мб, webm,
1280x720, 1:29
409 424894
>>4893
Напомнило вебм
410 424896
>>4762
Взяли уравнения максвелла, и решили для "заряд вращается вокруг другого". Вышли волны и энергия излучалась.
874e2ba2672b9c3429450a66704b355e.png42 Кб, 613x542
411 424898
Помогите, пожалуйста, супердолбоебу. Я, кажется, зря списывал матешу на первом курсе, теперь не могу понять, как получилось такое преобразование скорости.
А без понимания этого я не смогу сдать курсач и написать защиту по нему.
Где я наебался?
412 424903
>>4898
Что тебе непонятно? d(v2)/2dt = vdv/dt.
Базовая же хрень из школьной программы. (x2)' = 2x
413 424904
>>3747 (OP)
А вот есть там гормоны удовольствия, зоны удовольствия там в мозге, а есть ли гормоны страданий? зоны страданий?
414 424905
>>4903
Вот ето и не понятно, общую формулу дифференцирования забыл)0
415 424906
>>4903
Но ведь степнь числа не эквивалентна числу умноженному на себя раз равное величине степени. просто если V в степни 2 равно V умножить на V, то чему равно V в степни n например.
416 424907
>>4904
Есть.
417 424908
>>4906

>то чему равно V в степни n например.


nVn-1
418 424909
>>4898
пиздец... докатились
419 424911
Часто встречаю слово "спектр" в различных областях науки. Как его правильно понимать ? Распределение чего-то по чему-то или разложение по базису(что есть тоже распределние чего-то по базисным функциям) ?
420 424912
>>4911
Два основных значения, в зависимости от области.

Наиболее вероятное, спектр -
это распределение интенсивности по частотам (сюда же спектрограмма). От спектра mp3 файла до спектра излучения водорода, всё входит в это понятие. Сигнал раскладывается на частоты (точнее, диапазоны частот), и мы смотрим, что сильнее ебошит. Так просто удобней изучать всё волновое-периодическое.

Второй вариант, более строгий математически. Спектр оператора - это фактически дополнение его резольвентного множества. В более простом случае конечномерных операторов, спектр это просто собственные числа соответствующей матрицы. В частном случае применения функционального анализа в квантовой механике, квантовые наблюдаемые при измерении могут принимать значения только из своего спектра.
421 424915
>>4908
И нахуй ты всрал сюда степень? лол.

>Вырази мне степень возведение в степень через умножение


>вот выразил через возведение в степень


С таким же успехом ты мог бы хуй соснуть и сказать "выполенно", в прочем ты и так его соснул.
422 424916
>>4915
Лол, ну ты тупенький
V^0 = 1
и в итоге твое манячисло схлопнется в ряд произведений.
Можно записать как
П(от 1 до n) V(i) где V(i) = V
Если тебе так бомбит от степеней.

Мимо
423 424917
>>4906
Что за хуйню ты пишешь?
424 424918
>>4916
wat?
8e21415607c90973c17bcb2be2a443e1.png18 Кб, 981x258
425 424920
>>4903
У меня еще вопрос, помоги пожалуйста. Какую формулу применять для такого рода дифференцирования?
426 424921
>>4920
Забыл дописать, по правилу дифференцирования произведения?
(f∙g)' = f'∙g + f∙g'.
427 424922
>>4921
Во второй формуле все три слагаемых неверны. Такое чувство, что ты просто на дт разделил.

d(fg) =gdf+fdg
В частности, d(f^2)=d(ff)=fdf+fdf=2fdf
428 424923
>>4922
Я вообще что-то охуеваю. Откуда эти "люди", тут берутся? Это школьники? Почему не в un тогда? Зачем вообще пытаться заниматься физикой,если не знаешь простейшего матана? Почему не пойти ботать матан?
429 424924
>>4922

>на дт разделил


А то. Перехожу от интегрального вида к дифференциальному.

>В частности, d(f^2)=d(ff)=fdf+fdf=2fdf


Ну да. Сначала я дифференцирую скорость, потом полученное 2fdf делится на dt, получаю V*a. Просто не доделал дифф-е числителя.
Благодарочка за обьяснение. Этих знаний мне хватит чтобы закрыть бакалавриат)0
430 424925
>>4923
Это студенты. Судя по материалу - второй курс, третий семестр. Матан им должны были преподавать на первом курсе. Но ты знаешь, его все списывают, спрашивал знакомых из 10 разных шараг, в том числе МГТУ, РУДН, МЭИ, МГСУ, всюду списывали. Знакомых из МГУ и МФТИ нет, к сожалению.
431 424926
>>4925
Материал не второго курса. На втором уже всё в Гамильтовой механике обмазано и скобками Пуассона.
432 424927
>>4926
Это теоретическая механика, батан.
433 424928
>>4927
В любом случае механика на первом курсе.
434 424930
>>4924
В изначальной картинке ты написал какую-то хуйню. Скобки у тебя аки аниму волшебницы обладают таинственной силой. Будем считать, что это очепятка, хотя практика преподавания мне подсказывает, что ты в этой хуйне плаваешь.
435 424931
>>4928
Зависит от учебной программы. На первом преподают аналитическую механику.
>>4930
Еще как плаваю, братиш, а уж как я в прошлом году решал ДУ второго порядка... Хочу забыть как кошмарный сон. Хотя это надо будет по учебе дальше.
436 424932
>>4930
Я тоже угарнул с его dx в скобке. Человек на заочке видимо учится, и ему диплом нужен. А не "цифры писать, да буковы".

Кстати, анончиги. Кто-нибудь шарит в КЭД? Я сам нихуя не могу разобраться. Возможно я просто тупой и не умею воспринимать тензорные формулы, возможно я просто даун.
437 424933
>>4931
Анон, удачи, конечно, но это реально простые вещи, которые выступают в роли букв/слов естественнонаучного языка. Практически вся наука изучает изменение, понимание производных должно быть на уровне понимания сложноподчиненных предложений или основ логики и импликации.

>>4932
Я тебе советую почитать англоязычную литературу, гуглишь qed for undergraduates. В чём именно проблемы? Можешь почитать квантовую теорию поля вобщем, а потом перейти к частному случаю кэд. Peskin/schroeder считаются стандартом, но если у тебя проблемы с тензорами, то это оверкилл. Например, zee, или klauber более казуальные. Тензоры в ктп/кэд пришли из СТО, возьми почитай её (скажем, schutz, или caroll, или misner thorne wheeler).
438 424936
>>4918
Если у тебя n целое положительное число(натуральное число), то:
V^n = умножению V на себя n раз, есть такой оператор греческая П, это как сигма(т.е. оператор суммы) только он не складывает, а умножает числа в ряде.

А вообще есть более общая абстракция, гугли стрелочную нотацию кнута.

Например точно так же умножение можно заменить сложением
т.е. V*n это проссумировать V n раз
твои степени т.е. V^n это умножить V n раз
тетрация - это когда V^(V^(V^V) и так далее n раз, такая пирамидка из степеней. это то же самое что возвести в V в степень V, потом возвести V в степень числа, полученного на предыдущем этапе и так далее n раз.

Ну и по аналогии для более высоких степеней.
439 424937
Перед сном выплеснулось очень много адреналина и я выпил таблетку для переваривания. В итоге всю ночь фонтанировал говном и блевал аки кот. Из-за чего?
440 424938
>>4936

>высоких порядков


фикс
441 424963
>>4923
учитывая, что производные еще в 10 классе изучаются, хз
442 424964
>>4932
на уровне теории поля или ктп?
443 424965
>>4932
вообще есть очень хорошая книга по теории поля степаньянца "классическая теория поля". Все очень разжевано и понятно.
8.png128 Кб, 1492x848
444 424968
Инженер обращается за помощью к математикам. Я не прошу вас искать решение, просто скажите возможно оно или нет.

<--------Внимание на экран
Снимок экрана от 2017-11-15 02:29:08.png18 Кб, 602x247
445 424970
Как такое форматирование в латексе сделать чтобы было выравнивание по левому краю без отступов при переходе на новую строку?
446 424983
>>4970
Тут нужен код, а то может ты как последняя пидораха eqnarray пользуешься, кто тебя знает.
447 424984
>>3747 (OP)
Возможно не в тему, но всё же. Ребят, можете подсказать видео лекции или книги, желательно от Русских учёных, объясняющих устройство зрения в сумме с мозгом. Просто тут мне Шадрина посоветовали, но его невозможно слушать, мне хочется услышать сухими фактами объяснение, а не рассусоливание на тему. Могу быть не прав.
448 424985
>>4924

>1


Соус?
449 424988
>>4983
Зачем код? Просто скажи куда выражение писать чтобы переходы так выравнивались.
450 425003
Почему микроволны опаснее света? У них же частота меньше должна быть не?
451 425005
>>5003
А они опаснее? а частота меньше?
452 425006
>>5003
Не опаснее. Не меньше.
453 425007
А электроны вращаются по или против часовой стрелки?
454 425008
>>5007
Хех... они никак не вращаются. У водорода в основном состоянии вообще момента нет.
455 425009
>>4968
Не всё понятно.
Ну, смотри, для минимума и максимума надо искать нули производной. Только производная будет по x, а интеграл у тебя по s. Если выразить s через x, то может что-нибудь получиться. Если s от x не зависит, то больше похоже на жопу.
Мне кажется, тут лучше порешать численно.
456 425011
>>5009
ds=dx
численно муторно, хуёвая архитектура software для задачи в разы комплексней чем на рисунке. Мне нужен какой то математический инструмент, а не тупой подбор. Конечно у меня есть интуиция.
457 425013
Все аффинные преобразования кроме зеркального отражения могут быть выражены в матричном виде, да или нет ? Поворот квадратной матрицы не может быть записан в матричном виде, если да то как ?
458 425016
>>5013

>Все аффинные преобразования кроме зеркального отражения могут быть выражены в матричном виде, да или нет ?


Вообще все аффинные преобразования без исключения так представимы. Аффинная группа изоморфна подгруппе полной линейной группы матриц.

>Поворот квадратной матрицы


Это что-то магическое. Первый вариант - ты очепятался/ошибся, поясняй. Второй вариант - ты реально имел ввиду поворот как преобразование над вектором пространства матриц (хотя понятие поворота некорректно пока ты не ввёл метрику на пространстве матриц). Тогда, действительно, можно любое линейное преобразование над матрицей записать в виде матрицы.
459 425020
>>5016

>все аффинные преобразования без исключения так представимы.


Т. е. и симметрия относительно оси и относительно точки ?

>Это что-то магическое.


a b поворот на 90 c a
c d b d
Это из обработки изображений, ведь оно матрица чисел(для простоты 256 градаций серого) и его можно повернуть на 90, 180 и 270. Можно также и зеркально отразить относительно осей и диагоналей. В данном случае отражение точно нельзя представить матрицей, а поворот можно ?
460 425021
>>5020
Я забыл что двиг двача жрёт пробелы, давно не баловался с треугольником Серпиньского.
a b
с d
Должно стать
c a
d b
461 425023
>>5020
матричное представление S_4 не катит что ли?
462 425024
>>5020
Всё можно представить)
463 425025
>>5020

>Т. е. и симметрия относительно оси и относительно точки ?


Да

>a b поворот на 90 c a


>c d b d


Анон, изъясняйся по-человечески. Точно тут буквы не напутал?

>>5021
Теперь ясно. И это скорее поворот на -90, ну ладно.

>ведь оно матрица чисел


Так-то так, но числа в матрице приходят не из ниоткуда, а как представление линейного оператора.

Ответ на твою задачу прост - повороту на минус 90 градусов матрицы A соответствует действие A'B, где штрих - транспонирование, B - матрица [0 1; 1 0], и подразумевается умножение матриц.

Если ты хочешь это действие записать без транспонирования, то это тоже возможно. Точно также возможно и представить отражение, так что ответ на твой вопрос

>данном случае отражение точно нельзя представить матрицей


тоже можно.

Как я писал в предыдущем посте, ты действительно рассматриваешь множество матриц само как векторное пространство. Тут "поворот" и "отражение", математически, суть просто перестановки. Значит, это элементы симметрической группы Sn (точнее, S n^2), а они представимы в виде матрицы.

Существует много представлений, можно использовать стандартное или естественное. Например, естественное будет выглядеть как n^2 x n^2 матрицы, действующие на твою "матрицу", которая запишется в виде столбца n^2 x 1.
463 425025
>>5020

>Т. е. и симметрия относительно оси и относительно точки ?


Да

>a b поворот на 90 c a


>c d b d


Анон, изъясняйся по-человечески. Точно тут буквы не напутал?

>>5021
Теперь ясно. И это скорее поворот на -90, ну ладно.

>ведь оно матрица чисел


Так-то так, но числа в матрице приходят не из ниоткуда, а как представление линейного оператора.

Ответ на твою задачу прост - повороту на минус 90 градусов матрицы A соответствует действие A'B, где штрих - транспонирование, B - матрица [0 1; 1 0], и подразумевается умножение матриц.

Если ты хочешь это действие записать без транспонирования, то это тоже возможно. Точно также возможно и представить отражение, так что ответ на твой вопрос

>данном случае отражение точно нельзя представить матрицей


тоже можно.

Как я писал в предыдущем посте, ты действительно рассматриваешь множество матриц само как векторное пространство. Тут "поворот" и "отражение", математически, суть просто перестановки. Значит, это элементы симметрической группы Sn (точнее, S n^2), а они представимы в виде матрицы.

Существует много представлений, можно использовать стандартное или естественное. Например, естественное будет выглядеть как n^2 x n^2 матрицы, действующие на твою "матрицу", которая запишется в виде столбца n^2 x 1.
464 425026
Частный случай в тригонометрии гласит:
sin x = 1
x = π/2 + 2πk где k ∈ Z.
Вопрос вот в чем: если "+ 2πk" нам необходимо тупо чтобы наматывать круги по числовой окружности, возвращаться в ту же точку и не упускать корни уравнений, то почему мы идем только против часовой стрелки? Ведь если мы пойдем на круг назад (т.е. "- 2πk"), то тоже окажемся в той же точке.
Почему в таком случае формула не выглядит так:
x = π/2 ± 2πk где k ∈ Z. ?
Number-line.svg.png6 Кб, 750x50
465 425027
>>5026

>Почему в таком случае формула не выглядит так


Она так и выглядит, просто ты не понимаешь что Z это множество целых чисел
466 425035
Кто сказал фразу "Существует позорное явление под названием математическая статистика"?
467 425037
>>5035

>Кто сказал фразу


Долбоёб, не разбирающийся в статистике. Одно из самых распространённых явлений - какой-нибудь социолог или экономист, который не имеет математической базы, но вынужден использовать матстат, и неверно интерпретирует результаты.
468 425039
Аноны, Еськов все еще в тренде? И если нет, то что еще из актуального научпопа по возникновению жизни можно почитать?
469 425045
Возможно вопрос уровня школьной программы, но все же хотелось бы видеть ответ на этот глупый вопрос. Я решил по экспериментировать с теоремой пифагора. Я взял прямоугольный треугольник с a=1 и b=2 и с неизвестная гипотенуза. В треугольнике ABC, угол C равен 30 градусом и противолежит стороне a. Сначала я заюзал a^2+b^2=c^2, что будет 1^2+2^2=5. с^2=5, значит с=√5. Приблизительное значение √5 это 2,236
с≈2,236
Не забываем, что у меня был угол С в 30 градусов противолежащий а. Значит с=a*2(т.к c гипотенуза), то есть с=1+1; с=2 ,но как так? 2.236≠2
Я не понимаю, почему так произошло возможно я где-то ошибся или что-то не понимаю. Возможно такое действие является невозможным.(Я просто школьник и ничего не смыслю, разъясните если я допустил грубейшую ошибку, чтобы я больше так не делал. Задал я вопрос чисто из любопытства, ибо интересно очень.)
470 425046
>>5045
Почему 30
471 425047
>>5046
Это происходит если имеется условие в виде угла в 30 градусов противолежащий стороне а
472 425048
>>5047
Ты не понял. Не могут быть стороны 1 и 2 и угол 30.
473 425049
>>5048
Ну я и просил разъяснить. Ибо я по идее не вижу ошибок и нужен взгляд с другой стороны, к тому же я всего-лишь школьник и мало что смыслю.
474 425051
>>5049
Пифагора теорема работает только в прямоугольных треугольниках. Ты взял наобум треугольник, и проебался. 30 градусов, это когда противолежащая сторона в два раза меньше гипотенузы. Ты же взял отношение катетов за 2.
475 425052
>>5051
Но ведь могут быть прямоугольный треугольник с углом в 30 градусов.
476 425053
>>5052
Может. Но тогда если катет против него равен 1. то второй катет должен быть равен 31/2
477 425054
>>5049
В треугольнике 6 величин (3 угла, 3 длины сторон). Но они не являются независимыми: если ты напишешь 6 (5,4) рандомных чисел, они могут и не соответствовать никакому треугольнику. Оказывается, трёх величин хватает, чтобы задать треугольник. Например, две стороны и угол.

Твоё начальное условие уже включает три величины - угол 90г, сторона 1, сторона 2. Этого уже достаточно, чтобы однозначно определить треугольник. Дальше ты вводишь свои 30г, это дополнительное условие, четвертое число. Но треугольник-то у тебя уже есть, маловероятно, что ты угадал один из углов. И действительно, у твоего треугольника другие углы.

Вобщем, все из-за твоих 30г,которые я так и не понял, откуда ты взял, отчего и спросил сразу же в предыдущих постах.
478 425055
>>5054
Я не до конца понимаю, что вы имеете ввиду. Я такой человек, мне требуются подробнейшие объяснения. Ибо я достаточно глупый и не понимаю что-то, если пояснения недостаточно подробны.
479 425056
>>5055
Можно вопрос не по теме. Чисто для себя, как для преподавателя интересно. В какой момент ты вообще перестал слушать или понимать что происходит на уроке? И почему?
480 425057
>>5053
Любопытно, что я опять столкнулся с этой проблемой.
481 425058
>>5057
Ага, я понял, теперь я решил мелкую проблему для меня появившиеся из-за недостатка знании.
482 425059
>>5058
Хотя не, все равно жопа
483 425060
>>5058
не в знаниях дело. Не совершай эту ошибку. Многие после ошибки, считают что "я не знал чего-то" и это пускает их еще в более неверную сторону.
484 425061
Я уже ничего не понимаю, что не так с этой великой теоремой(9((
485 425062
>>5061
Траллишь?
486 425063
>>5060
Но если я что-то не знаю, я не смогу применять знания практике т.к их не будет.
487 425064
>>5063
Но теорему пифагора ты знаешь. Больше тут ничего и не надо.
488 425065
>>5064
И теперь она вызывает у меня парадокс в моих руках, какой я не возьму прямоугольный треугольник с 30 градусами противолежащий стороне а, то опять эта проблема.
489 425066
>>5065
А какой ты берешь?
490 425067
>>5066
Да любой могу.
491 425068
>>5067
Кек... любой прямоугольный треугольник с углом в 30 градусов подобен любому другому с углом 30. Так что про "любой" ты немного зря.
492 425069
>>5068
Я вроде уточнил что любой прямоугольный с углом в 30 градусов, то есть я имел ввиду, что я по большей части менял лишь длину сторон.
493 425070
>>5069
Хорошо... смотри.
Я беру и говорю. Пусть у меня в треугольнике
1) угол 90
2) еще один угол 30
3) катет 1
Всё... треугоьник готов, у всего уже есть конкретные величины, я не могу сказать что гипотенуза равна 3, или другой катет равен 2. Так как треугольник уже готов.
или
1)угол в 90
2)один катет 1
3)второй катет 2
Все, треугольник готов, и угла в 30 градусов у него нет, я не могу уже ничего определить.
494 425071
>>5070

>доопределить


так как расчитать все углы и стороны спокойно
495 425072
>>5071
То есть я к треугольнику могу дать только три условия и все? А что если я найду угол в 30 и затем проделаю, те же действия, что я делал и выше?
496 425073
>>5072
Да, только три условия.
сторона и два угла
две стороны и угол
или три стороны. Больше вариантов нет.
497 425075
>>5045

>a=1 и b=2


Это тип катеты ?
Тогда с = корень из 5, по Пифагору. И углов в 30 градусов быть уже не может. Или ты берёшб катеты 1 и 2 и угол в 30, тогда другие углы уже не могут быть 90 и Пифгор не действует.
498 425088
Если взять очень мощный лазер и посветить им на луну, проведя по ней очень быстро, то луч по поверхности будет двигаться быстрее скорости света, разве нет?
499 425091
>>5088
Пошел ты нахуй.
мимо-Эйнштейн
А так нет. Лазер это узконаправленные пучки света. Можешь провести аналогию с пульверизатором или шлангом воды, когда поворачиваешь его, струя изгибается. Так и лазер изогнется. Вот если бы ты достал цельный стержень, и тыкнул бы им в Луну, вот тогда бы Эйнштейн пошел бы ебать твою мамашу, но тебе пока что повезло
500 425092
>>5091
Ну ок, беру абсолютно твердый и суперлегкий стержень длиной в хуймиллиард километров и машу им. Скорость на конце будет выше скорости света, так?
cef338b94b3d6c52690c3ad44d7c71ab.png281 Кб, 1282x700
501 425094
>>5092
Ну все, посан, я иду к твоей мамаше.
Эйнштейн
502 425095
>>5011
В таком случае все проще. Можешь ещё пояснить, где у тебя константы, а где функции. Вот все эти M1 и прочее.
503 425096
Можно ли сделать какое-то следствие о физических особенностях вещества основываясь только на электронной конфигурации? Ну пример, дана электронная конфигурация ртути и как из нее сделать вывод, что ртуть - жидкий металл в домашних условиях, в не твердый например? Или вообще газ?
504 425105
>>5092
Да это хуита. Берем тело отрицательной массы. Оно разгоняясь выделяет энергию. Сначала собираем её в электричество, потом запускаем в Йеллоустоун. Пиндосы соснули, у нас халявная энергия.
505 425116
>>5025

>Если ты хочешь это действие записать без транспонирования, то это тоже возможно.


Попробовал записать транспонирование в матричном виде. Всё что смог придумать это представить матрицу как вектор-столбец из n элементов и умножить на матрицу из n^2 элементов, которая есть матрица перехода в другой базис, в котором вектора те же самые, но в другом порядке. Полагаю можно и лучше ?
507 425119
>>5096
Можно. Только для этого тебе понадобится либо охуенный опыт. Типа "я знаю, что 6s электроны, из-за релятивистких эффектов поджаты к ядру, поэтому ртуть должна быть более инертной и хуже образовывать связи, возможно она жидкая". Либо мощный комп, который может посчитать.
508 425136
>>3844

22 км это относительно самой низкой точки (на Марсе нет океанов). На Земле же самой низкой точкой является Марианская впадина с мощностью около 12 км. В итоге получается что на Марсе и на Земле самые высокие точки выступают над поверхностью земли примерно примерно на одном уровне .
8301-NTkzNmU4MDZjYQ.jpg126 Кб, 600x415
509 425242
>>5011
Функции:
Mcp, dir : Pmt(x) - константа. zp(x) - функция, которую ищем.
Mcp, ind : вычесляется при известной функции Mcp, dir и M1
Mcp : Mcp, dir + Mcp, ind
M1 : Задаётся юзером. Известная функция (часто линейная как на картинке, но может быть и кубической и квадратной)

Константы:
Pmt(x)
EI
510 425243
climate-change.jpg308 Кб, 1920x1080
511 425264
Реально ли антропогенное изменение климата или это все происки жидов выбивание грантов климатологами?
512 426433
>>5242
бамп
Тред утонул или удален.
Это копия, сохраненная 17 февраля 2018 года.

Скачать тред: только с превью, с превью и прикрепленными файлами.
Второй вариант может долго скачиваться. Файлы будут только в живых или недавно утонувших тредах. Подробнее

Если вам полезен архив М.Двача, пожертвуйте на оплату сервера.
« /sci/В начало тредаВеб-версияНастройки
/a//b//mu//s//vg/Все доски